Updated Feb 2025
I hope you enjoy reading this blog post.
If you want my team to help you with your USMLE preparations , click here.
Updated as of Feb 2025
Here are the detailed explanations of the new Free 120 NBME STEP 2 Sample Test Questions so you don’t waste your time trying to find the correct answers and explanations. We had our expert USMLE tutors who scored 260+ on their exams refine these answers and explanations for you so you can spend your valuable time learning!
You can download the Free 120 STEP 2 Sample Test Questions on the USMLE website here. The order of answers and explanations here is based on the online NBME platform (not the pdf) as of February 2025.
Table of Contents
- Block 1
- Block 2
- Block 3
Ace Your Step 2 CK with Expert Guidance
Struggling with Step 2 CK preparation? Get personalized 1-on-1 coaching from top scorers and experienced mentors.
Book a Session Today!
Trustpilot
Trustpilot
Block 1
Question 1:
The correct answer is B. This patient presents with sudden onset of flank/lumbar pain, gross hematuria, and unilateral hydronephrosis with hydroureter, strongly suggesting a ureteral stone. Non-contrast CT of the abdomen and pelvis is the gold standard for diagnosing ureteral stones. It provides a detailed view of the renal tract, identifying stones and their exact location, as well as any complications like obstruction.
Incorrect answers:
A. Captopril renography: Typically used for diagnosing renovascular hypertension due to renal artery stenosis, which is unrelated to this patient’s acute presentation.
C. MAG-3 renal scan with furosemide: Utilized to distinguish obstructive from non-obstructive hydronephrosis but is not indicated as the first-line test in acute obstruction suspected due to ureteral stone.
D. Radionuclide cystography & E. Retrograde pyelography: These are used for assessing vesicoureteral reflux and other structural abnormalities. They are not first-line for evaluating suspected stones.
F. Ultrasonography of the spinal column: Indicated for spinal dysraphism, often identified by external markers such as a tuft of hair or dimple in the lower back, which are absent in this case.
G. Voiding cystourethrography: Performed to assess vesicoureteral reflux, which commonly presents with recurrent urinary tract infections, not acute flank pain and hematuria “HY”.
Question 2:
The correct answer is F. This patient’s symptoms of numbness, absent deep tendon reflexes, and decreased vibration and proprioception are consistent with peripheral neuropathy, likely due to vitamin B12 deficiency. Chronic calcium carbonate use can impair B12 absorption by reducing gastric acid secretion, which is required to release B12 from food. Vitamin B12 supplementation is necessary to prevent further neurological damage and to restore B12 levels. This is particularly important in elderly patients or those with long-term antacid use.
Incorrect answers:
A. Alcohol cessation: Chronic alcohol use can contribute to neuropathy but typically also causes cerebellar dysfunction, leading to ataxia and an abnormal gait, which are not observed in this patient.
B.Calcium carbonate cessation: Stopping calcium carbonate may reduce B12 depletion, but replenishing B12 stores with supplementation is the more direct treatment.
C.Folic acid supplementation: Folic acid deficiency causes macrocytic anemia but does not typically result in peripheral neuropathy.
D.Niacin supplementation: Niacin deficiency presents as pellagra, characterized by diarrhea, dermatitis, and dementia, not peripheral neuropathy.
E.Vitamin B1 (thiamine) supplementation: Thiamine deficiency results in Wernicke encephalopathy, presenting as confusion, ataxia, and ophthalmoplegia, which are not seen in this case.
Unlock your research potential with our dynamic course designed for medical students and graduates! Built by an experienced clinical researcher with 150+ publications and 3000+ citations!
Sign up NOW!
Question 3:
The correct answer is D. This patient presents with acute onset of cough, hoarseness, inspiratory stridor at rest, and moderate respiratory distress following a recent upper respiratory tract infection, which is consistent with croup. Croup is commonly caused by the parainfluenza virus and results in subglottic airway edema. Nebulized epinephrine is indicated for severe croup, characterized by stridor at rest and the use of accessory muscles for breathing (e.g., suprasternal and subcostal retractions). It works by reducing airway edema and relieving obstruction.
Incorrect answers:
A.Intravenous dexamethasone & F. Oral prednisone: Steroids like dexamethasone are effective for reducing airway inflammation but act more slowly. They are used as adjunctive therapy rather than initial treatment in severe cases.
B.Nebulized albuterol: Albuterol is used for bronchospasm in asthma, not for airway edema seen in croup.
C.Nebulized budesonide: Used for the chronic management of asthma, not acute treatment of croup.
E.Oral albuterol: Not commonly used in clinical practice due to limited efficacy and potential side effects.
G. Subcutaneous epinephrine: Indicated for anaphylaxis and angioedema, not for croup.
Question 4:
The correct answer is D. This patient is in the end stage of a terminal illness with acute respiratory failure. He has verbally expressed his desire to avoid mechanical ventilation, as confirmed by his wife. Even without formal documentation, verbal statements regarding medical care preferences are ethically and legally significant, especially when corroborated by close family. Respecting the patient’s autonomy and prioritizing his wishes aligns with ethical medical practice. Providing palliative therapy ensures comfort and dignity in his remaining hours.
Incorrect answers:
A.Perform endotracheal intubation and begin mechanical ventilation: Violates the patient’s expressed wishes and autonomy.
B.Perform endotracheal intubation and then consult the hospital ethics committee: Unnecessary because the patient’s wishes are clear and validated by a close family member.
C. Perform endotracheal intubation only: Contradicts the patient’s stated preference against mechanical ventilation.
E.Seek a court order to assign a legal guardian: Impractical for an acute situation requiring immediate action and unnecessary when the patient’s wishes are known.
Our Clients’ Success Speaks to Our Premium Service!
The Match Application Packages
Question 5:
The correct answer is D. This child presents with specific difficulties in reading despite having an average IQ and age-appropriate daily living skills, which is characteristic of a learning disorder. Learning disorders affect specific academic skills (e.g., reading, mathematics, or writing) and are not associated with global intellectual impairment. The slow acquisition of English is expected given his recent adoption and prior Russian language background. His receptive language abilities and age-appropriate skills further confirm that this is a specific academic issue rather than a global developmental delay. In adopted children, language acquisition difficulties are common due to exposure to a new language environment, and these must be distinguished from true learning disabilities by assessing their native language proficiency and academic skills.
Incorrect answers:
A. Autism spectrum disorder: This child does not exhibit persistent deficits in social communication or interaction, nor restricted or repetitive patterns of behavior.
B.Fetal alcohol syndrome: The patient lacks the characteristic facial features (e.g., smooth philtrum, thin upper lip), growth restrictions, and CNS abnormalities associated with this condition.
C.Intellectual developmental disorder: The child’s average IQ and age-appropriate adaptive functioning exclude this diagnosis.
E.Post-traumatic stress disorder: Although the child has a traumatic background, there are no symptoms of re-experiencing, avoidance, or hyperarousal to suggest PTSD.
F.Reactive attachment disorder: The child demonstrates healthy emotional attachment and social engagement, which excludes this diagnosis.
Question 6:
The correct answer is F. The presence of a raised, fleshy lesion on the vulva in an elderly woman raises suspicion for vulvar cancer. The definitive method for diagnosing such a lesion is through histological examination via biopsy. Risk factors for vulvar cancer include advanced age, chronic lichen sclerosus, human papillomavirus (HPV) infection, and smoking. Early biopsy is crucial for identifying malignancy and initiating treatment promptly.
Incorrect answers:
A.Cytologic evaluation of the vulva: Cytology is not sufficient for diagnosing vulvar cancer; biopsy is the gold standard.
B. CT scan of the abdomen and pelvis: Imaging is used for staging after a diagnosis od cancer, not as an initial diagnostic step for a vulvar lesion.
C.Application of an antifungal cream: There is no evidence of fungal infection, such as erythema or pruritus.
D.Application of a corticosteroid cream: The lesion does not exhibit signs of inflammation or conditions like lichen sclerosis which presents as a white patch.
E.Colonoscopy: used for cervical cancer assessment. This is unrelated to the presentation of a vulvar lesion.
Question 7:
The correct answer is C. This patient presents with clinical signs of peripheral arterial disease (PAD), including absent pedal pulses and a femoral bruit. Dyslipidemia is a significant and modifiable risk factor for PAD, making lipid profile assessment crucial in this scenario. PAD is a strong predictor of systemic atherosclerosis, necessitating aggressive risk factor management, including lipid-lowering therapy, to prevent cardiovascular events. Identifying and managing dyslipidemia helps reduce cardiovascular morbidity and mortality associated with PAD.
Incorrect answers:
A.Pentoxifylline therapy: Indicated for symptomatic intermittent claudication in PAD but unnecessary for asymptomatic patients.
B.Peripheral artery catheterization: An invasive procedure used for patients undergoing revascularization for acute limb ischemia, not for initial evaluation.
D.Warfarin therapy: Warfarin is used for anticoagulation in conditions like atrial fibrillation or venous thromboembolism but is not indicated for PAD. Instead, antiplatelets are used for PAD.
E.No further management is indicated: The patient requires further evaluation and management of PAD risk factors; neglecting this would miss an opportunity to prevent progression and associated complications.
Our Clients’ Success Speaks to Our Premium Service!
Our Interview Preparation Packages
Question 8:
The correct answer is C. This patient’s fever, sore throat, and significant neutropenia (marked decrease in white blood cells with relative neutropenia) are consistent with drug-induced agranulocytosis, a known adverse effect of trimethoprim-sulfamethoxazole (TMP-SMX). Agranulocytosis is characterized by an acute reduction in circulating granulocytes, predominantly neutrophils, and is often associated with infections such as pharyngitis due to the impaired immune response. TMP-SMX-induced agranulocytosis typically resolves after discontinuation of the drug.
Incorrect answers:
A.Acute mononucleosis: Typically presents with fever, sore throat, and lymphocytosis with atypical lymphocytes, not neutropenia. In addition, splenomegaly and cervical lymphadenopathy are expected.
B.Acute myelogenous leukemia: Presents with pancytopenia and immature blast cells on the peripheral smear, which are absent here.
D.Allergic reaction to lisinopril: Lisinopril is associated with angioedema and cough but does not cause neutropenia.
E.Myelofibrosis: A chronic condition leading to pancytopenia, splenomegaly, teardrop cells on the smear, and bone marrow fibrosis, which are not seen in this case.
F.Sepsis syndrome: Sepsis can cause leukocytosis or leukopenia, but the temporal relationship with TMP-SMX use strongly suggests drug-induced agranulocytosis rather than sepsis.
Question 9:
The correct answer is B. This patient with Parkinson’s disease is experiencing postural instability, evidenced by frequent falls, backward falls when walking, and failure to maintain posture during a pull test. Physical therapy, particularly focusing on balance training and gait exercises, is the most effective intervention to reduce the risk of future falls. Physical therapy improves strength, stability, and movement coordination, which are critical for addressing postural instability in Parkinson’s disease.
Incorrect answers:
A. Biofeedback: may be used for somatic symptom disorder, no consistent evidence supports its effectiveness in managing postural instability related to Parkinson’s disease.
C.Pramipexole therapy, D. Ropinirole therapy, E. Rotigotine therapy: dopamine agonists used for Parkinson’s symptoms of rigidity and bradykinesia but do not address postural instability or fall prevention.
Question 10:
The correct answer is C. The patient’s right knee examination shows no evidence of significant injury and does not meet the Ottawa Knee Rule criteria for imaging “knee pain and any of the following:
1. Age ≥55 years.
2. Isolated tenderness of the patella.
3. Tenderness at the head of the fibula.
4. Inability to flex the knee to 90 degrees.
5. Inability to bear weight both immediately after the injury and for four steps in the emergency department”. In contrast, the right ankle examination reveals tenderness at the distal fibula and restricted range of motion, which meet the Ottawa Ankle Rule criteria for an X-ray to rule out a fracture “pain in the malleolar zone and any of the following:
1. Bone tenderness at the posterior edge or tip of the malleolus.
2. Inability to bear weight both immediately after the injury and for four steps in the emergency department.
The Ottawa Rules are a validated clinical tool to determine the need for imaging in acute knee and ankle injuries, helping reduce unnecessary X-rays. These rules emphasize clinical findings such as bone tenderness and the inability to bear weight.
Incorrect answers:
A.X-ray (Knee) & X-ray (Ankle): The knee findings do not meet Ottawa Knee Rule criteria, making an X-ray unnecessary.
B.X-ray (Knee) & No diagnostic testing indicated (Ankle): Overlooks the clear indications for imaging the ankle based on the Ottawa Ankle Rules.
D.No diagnostic testing indicated (Knee) & No diagnostic testing indicated (Ankle): Fails to address the concerning findings of distal fibula tenderness and limited motion in the ankle, which warrant imaging.
Would You Like Us to Help You Ace Your STEP Exams?
Learn More!
Trustpilot
Trustpilot
Question 11:
The correct answer is D. This patient presents with episodic headaches, palpitations, sweating, and hypertension, along with a 4-cm adrenal mass on CT imaging. These findings strongly suggest pheochromocytoma, a catecholamine-secreting adrenal tumor. The first step in confirming this diagnosis is measuring plasma free metanephrines or 24-hour urine fractionated metanephrines and catecholamines. Elevated levels confirm the biochemical diagnosis, guiding further management.
Incorrect answers:
A.24-Hour urine collection for measurement of vanillylmandelic acid and 5-hydroxyindoleacetic acid: Used to diagnose neuroblastoma (vanillylmandelic acid) or carcinoid syndrome (5-hydroxyindoleacetic acid), not pheochromocytoma.
B.Adrenal venous sampling: Reserved for differentiating primary hyperaldosteronism causes, such as adrenal adenoma versus bilateral adrenal hyperplasia.
C. Laparoscopic left adrenalectomy and right adrenal biopsy: Surgery is premature without biochemical confirmation of pheochromocytoma, as it requires specific preoperative management to prevent hypertensive crises.
E.Transsphenoidal hypophysectomy: Relevant for pituitary tumors, not adrenal lesions.
Question 12:
The correct answer is B. This patient presents with symptoms of hyperglycemia (polyuria, polydipsia, weight loss), a blood glucose level of 578 mg/dL, and an elevated hemoglobin A1c of 10.7%, consistent with diabetes mellitus. The rapid onset of symptoms, significant weight loss, and low BMI strongly suggest type 1 diabetes mellitus (T1DM) rather than type 2 diabetes mellitus. In T1DM, there is an absolute deficiency of insulin due to autoimmune destruction of pancreatic beta cells. Insulin therapy is required to manage hyperglycemia and prevent diabetic ketoacidosis.
Incorrect answers:
A.Glyburide therapy: Sulfonylureas stimulate insulin secretion from functional beta cells, which are absent in T1DM.
C.Metformin therapy: Primarily used for type 2 diabetes to reduce hepatic glucose production; ineffective in T1DM.
D.Pioglitazone therapy: A thiazolidinedione that improves insulin sensitivity; not suitable for T1DM as it does not address the lack of insulin.
E.Sitagliptin therapy: A DPP-4 inhibitor used for type 2 diabetes to enhance incretin activity; ineffective in T1DM.
Become a Researcher in the United States
No Experience Required! Just join the live & interactive workshop to learn all the secrets of landing a research position in the U.S
Enroll Here
Trustpilot
Trustpilot
Question 13:
The correct answer is B. This patient presents with severe shortness of breath, crackles extending halfway up the lung fields, low oxygen saturation, an S3 gallop, and hypotension shortly after a myocardial infarction. These findings are consistent with acute pulmonary edema due to acute left ventricular failure. The loss of functional myocardium from the myocardial infarction results in decreased cardiac output, increased left ventricular end-diastolic pressure, and pulmonary congestion.
Incorrect answers:
A.Acute dilation of the aortic root: Typically presents with severe chest pain and a widened mediastinum on chest X-ray, which is not described here.
C. Intravascular hypovolemia: Causes hypotension and tachycardia but is associated with dry mucous membranes and absent pulmonary edema.
D.New ventricular septal defect: Presents with a new loud holosystolic murmur best heard at the left lower sternal border and signs of left-to-right shunting, which are absent in this patient.
E.Ruptured papillary muscle: Leads to acute severe mitral regurgitation with a new loud holosystolic murmur and pulmonary edema, neither of which are noted in the patient.
Question 14:
The correct answer is E. This 12-year-old girl shows no signs of secondary sexual characteristics, but her presentation is not yet abnormal. Puberty typically begins between ages 8-13 in girls, and the absence of breast development (thelarche) by age 13 is the threshold for considering delayed puberty. This patient’s lack of breast development is still within the normal range for her age. Regular monitoring is sufficient unless she reaches age 13 without pubertal signs.
Incorrect answers:
A.Measurement of serum follicle-stimulating hormone and luteinizing hormone concentrations: There is no evidence of primary ovarian failure or hypothalamic-pituitary axis dysfunction requiring hormonal assessment.
B. Measurement of serum growth hormone and thyroxine concentrations: The patient’s growth and weight percentiles are normal, ruling out growth hormone deficiency or hypothyroidism.
C.MRI of the brain: There is no clinical suspicion of central causes of delayed puberty, such as a pituitary or hypothalamic tumor.
D. X-ray of the left hand and wrist to determine bone age: Bone age assessment is unnecessary unless there is significant growth delay or concern for abnormal skeletal maturity.
Get access to our list of Electives and U.S. Clinical Experience for IMGs!
Learn More
Question 15:
The correct answer is A. This randomized controlled trial (RCT) found that the new antiarrhythmic drug reduced recurrent episodes of ventricular tachycardia, but the investigators incorrectly concluded that it decreases mortality from cardiac arrhythmia.
This is an example of extrapolation beyond data—the study did not assess mortality as an outcome, so making claims about mortality reduction is scientifically invalid. When interpreting study results, it is crucial to distinguish between statistical significance and clinical relevance, avoiding unwarranted extrapolation of findings to outcomes not directly measured, such as mortality.
Incorrect answers:
B.Insufficient power – The study found a statistically significant difference (p < 0.05), meaning it had sufficient power to detect an effect.
C.No information regarding confidence interval – While confidence intervals provide additional precision, the main flaw here is the overgeneralization of results, not a lack of interval reporting.
D.Selection bias – There is no mention of inappropriate participant selection or non-randomized allocation, making selection bias unlikely.
Question 16:
The correct answer is E. The newborn hemoglobin electrophoresis showing an FS pattern indicates that the infant has sickle cell disease (SCD). The absence of hemoglobin A confirms this diagnosis. SCD predisposes children to life-threatening infections, particularly due to functional asplenia from early splenic infarction. The most appropriate next step is to initiate penicillin prophylaxis to prevent invasive infections, particularly from Streptococcus pneumoniae. Prophylaxis is typically started by 2 months of age and continued until at least age 5.
Incorrect answers:
A.Deferoxamine therapy: Used for iron overload in transfusion-dependent patients, not for routine management of SCD.
B. Hydroxyurea therapy: Increases fetal hemoglobin (HbF) and reduces vaso-occlusive crises but is not started in asymptomatic neonates.
C. Iron supplementation: Not indicated unless there is iron deficiency anemia, which is rare in neonates with SCD.
D.Monthly blood transfusions: Reserved for specific complications of SCD, such as stroke prevention, and is not routine at this stage.
F. Vitamin B12 (cyanocobalamin) supplementation: Not indicated, as there is no evidence of vitamin B12 deficiency.
What Do Our MATCH Application Packages Include?
Advisor UNLIMITED Access
We get how stressful the residency match process is, so we're here for you - communicate with your personal advisor ANYTIME you need!
Personal Statement Editing
Our editing includes not only language but also context, structure, and content advising.
ERAS Application Editing
The editing goes beyond language and grammar corrections to structure, design, and content based on your personal story and achievement.
Interview Preparation
The best way to learn something is to do it. That’s why we divide our interview preparation sessions into two parts.
Mock Interview + Feedback
Sign up NOW!
Question 17:
The correct answer is C. The patient has features consistent with DiGeorge syndrome (22q11.2 deletion syndrome), including congenital anomalies such as a cleft palate and ventricular septal defect, as well as recurrent infections (pneumonia and otitis media). A hallmark of DiGeorge syndrome is T-cell lymphopenia due to thymic hypoplasia or aplasia, which impairs the adaptive immune response and increases susceptibility to infections. A complete blood count in this patient would likely show lymphopenia secondary to the T-cell deficiency.
Incorrect answers:
A. Eosinophilia: Seen in parasitic infections, allergic reactions, and some autoimmune conditions, none of which are evident here.
B.Lymphocytosis: Typically occurs in viral infections such as infectious mononucleosis, which is not suggested by the history or presentation.
D. Neutropenia: More commonly associated with bone marrow suppression, congenital conditions like Kostmann syndrome, or aplastic anemia, none of which are relevant here.
E. Neutrophilia: Typical of bacterial infections or inflammatory conditions but does not explain the recurrent infections due to immunodeficiency.
Question 18:
The correct answer is E. The patient presents with hematemesis, mid-epigastric tenderness, and positive stool occult blood, which are consistent with an upper gastrointestinal (GI) bleed. Esophagogastroduodenoscopy (EGD) is the diagnostic and therapeutic procedure of choice for upper GI bleeding. It allows direct visualization of the esophagus, stomach, and duodenum, enabling identification and treatment of the bleeding source.
Incorrect answers:
A.Abdominal CT scan: Not first-line for GI bleeding; does not provide therapeutic capability.
B. Octreotide scan: Used for detecting ectopic endocrine activity, not for acute GI bleeding.
C. Technetium 99m scan: Used for slow or intermittent GI bleeds, but EGD is the preferred diagnostic tool for acute cases.
D.Colonoscopy: Evaluates the lower GI tract and is irrelevant for hematemesis, which indicates an upper GI source.
F.Mesenteric angiography: Reserved for cases of refractory bleeding when endoscopy is not diagnostic or therapeutic.
Boost your USMLE scores with personalized, one-on-one tutoring from our expert tutors at The Match Guy!
Learn More!
Trustpilot
Trustpilot
Question 19:
The correct answer is D. The patient has an upper GI bleed confirmed by esophagogastroduodenoscopy, and the endoscopic findings likely reveal an actively bleeding lesion or a high-risk lesion for rebleeding, such as a visible vessel or adherent clot. Endoscopic hemostatic therapy, which includes techniques such as injection therapy, thermal coagulation, or application of hemostatic clips, is the most appropriate next step. These interventions control active bleeding and reduce the risk of rebleeding.
Incorrect answers:
A. Octreotide therapy: Used for variceal bleeding associated with portal hypertension, not for non-variceal bleeding.
B.Omeprazole therapy: Proton pump inhibitors are used for peptic ulcer disease to reduce acid production but do not control active bleeding.
C.Tyrosine kinase inhibitor therapy: Indicated for gastrointestinal stromal tumors, not for acute GI bleeding.
E.Endoscopic biopsy: Used for diagnosing suspected lesions but is not the priority in acute bleeding.
F. Endoscopic resection: Appropriate for removing tumors or polyps but not for managing acute bleeding.
Question 20:
The correct answer is B. This patient presents with a persistent, nonproductive cough following an upper respiratory tract infection, accompanied by post-tussive vomiting and bursts of coughing, which are characteristic of pertussis (whooping cough). Bordetella pertussis, the causative organism, is treated with a macrolide antibiotic such as azithromycin. Early treatment reduces bacterial transmission, although it may not significantly alter the course of symptoms once the paroxysmal stage has begun.
Incorrect answers:
A.Albuterol: Used for bronchospasm in asthma or COPD, which this patient does not have.
C.Levofloxacin: A fluoroquinolone used for severe or complicated bacterial infections but not indicated for pertussis.
D.Omeprazole: A proton pump inhibitor for GERD, unrelated to this patient’s symptoms.
E.Prednisone: An anti-inflammatory for asthma, COPD exacerbations, or autoimmune diseases, none of which are present here.
Question 21:
The correct answer is B. This neonate has elevated TSH and low free thyroxine (FT₄), indicating primary hypothyroidism. Congenital hypothyroidism is most commonly caused by thyroid dysgenesis. Early initiation of levothyroxine therapy is critical to prevent irreversible intellectual disability and developmental delays. Treatment should be initiated as soon as possible to ensure normal thyroid hormone levels during this critical period of brain development.
Incorrect answers:
A. Hydrocortisone therapy: Used for adrenal insufficiency, not primary hypothyroidism.
C. Monthly thyroglobulin measurement: Thyroglobulin is not a primary marker for assessing thyroid function or guiding management.
D. Monthly TSH and FT₄ measurements: Monitoring thyroid function is important but does not replace the need for immediate treatment.
E. Radioactive iodine uptake scan: Contraindicated in neonates due to the risks associated with radiation exposure.
F. Ultrasonography of the thyroid gland: May identify structural abnormalities, but it is not required for initial treatment and does not influence immediate management.
Need Help Strengthening Your Research Experience?
Residency programs love applicants with strong research experience! Learn how to publish research, get involved in projects, and impress program directors with our comprehensive clinical research course.
Learn More!
Trustpilot
Trustpilot
Question 22:
The correct answer is D. This patient presents with progressive heart failure symptoms (dyspnea, orthopnea, pitting edema, S₃ gallop, pulmonary crackles) along with hypertension, exertional chest pain, and severe gingivitis (“meth mouth”), which are highly suggestive of chronic methamphetamine use.
Methamphetamine is a sympathomimetic stimulant that causes hypertension, tachycardia, and direct myocardial toxicity, leading to dilated cardiomyopathy and heart failure. Long-term use is also associated with dental decay and severe gingivitis, commonly referred to as ‘meth mouth’.
Incorrect answers:
A. Cocaine – Can cause myocardial ischemia and infarction, but is not strongly associated with dilated cardiomyopathy or gingivitis.
B. Heroin – More commonly leads to non-cardiogenic pulmonary edema, respiratory depression, and infectious complications rather than heart failure.
C. Methadone – Used for opioid dependence, associated with respiratory depression and QT prolongation, but not cardiomyopathy or gingivitis.
E. Toluene – Inhalant abuse can cause renal tubular acidosis and multi-organ damage, but it is not associated with heart failure or dental decay.
Question 23:
The correct answer is A. This patient presents with opioid overdose (lethargy, bradypnea), limb ischemia (cold, cyanotic extremity), and severe rhabdomyolysis (creatine kinase >50,000 U/L, hyperkalemia, metabolic acidosis). The USMLE/NBME emphasizes rhabdomyolysis as a common cause of myoglobin-induced AKI, making this the greatest risk.
Rhabdomyolysis leads to AKI through renal tubular obstruction, direct toxicity from myoglobin, and intravascular volume depletion. The elevated creatine kinase confirms significant muscle injury, and AKI is the most immediate and life-threatening complication in such cases.
Incorrect answers:
B. Acute liver failure – More commonly seen in acetaminophen overdose, not opioid toxicity.
C. Cardiac arrhythmia – Hyperkalemia from rhabdomyolysis can cause arrhythmias, but AKI is the primary risk.
D. Hypocalcemia & E. Hypophosphatemia – Electrolyte imbalances occur in rhabdomyolysis but are secondary to AKI and are not the greatest immediate risk.
Question 24:
The correct answer is E. This patient presents with chronic, intermittent non-bloody diarrhea, weight loss, and a history of drinking untreated freshwater from a spring-fed pond, which is highly suggestive of giardiasis.
Giardia lamblia is a protozoal parasite transmitted via ingestion of cyst-contaminated water, especially from rural or wilderness areas. It causes fat malabsorption, bloating, flatulence, and prolonged watery diarrhea. The presence of cysts or trophozoites on stool microscopy confirms the diagnosis.
Incorrect answers:
A. Campylobacter jejuni – Causes bloody diarrhea, often linked to undercooked poultry, not freshwater exposure.
B. Clostridioides difficile – Typically associated with recent antibiotic use, leading to pseudomembranous colitis.
C. Entamoeba histolytica – Causes dysentery (bloody diarrhea) and can lead to liver abscesses.
D. Escherichia coli – Various pathogenic strains cause gastroenteritis, but freshwater exposure is not a typical source.
F. Salmonella enteritidis – Associated with contaminated poultry/eggs, typically causing fever and bloody diarrhea.
G. Shigella dysenteriae – Produces severe dysentery, not prolonged, intermittent diarrhea.
Would You Like Us to Help You Ace Your Personal Statement?
Learn More!
Trustpilot
Trustpilot
Question 25:
The correct answer is E. This patient presents with acute unilateral pelvic pain, adnexal tenderness, and ultrasound findings of a simple ovarian cyst with free fluid, which strongly suggests an uncomplicated ruptured ovarian cyst.
Ovarian cyst rupture is common in reproductive-aged women, particularly mid-cycle, and results in mild-to-moderate pain that worsens with movement. The presence of free fluid on ultrasound is consistent with cyst rupture. In hemodynamically stable patients without signs of infection or ongoing bleeding, observation with pain management is the appropriate next step.
Incorrect answers:
A. Antibiotic therapy – Indicated for pelvic inflammatory disease (PID), but this patient has no fever, cervical motion tenderness, or purulent discharge, making infection unlikely.
B. Appendectomy – Acute appendicitis typically presents with fever, nausea, and right lower quadrant pain, not left adnexal tenderness.
C. CT scan-guided aspiration – Used for draining abscesses or evaluating uncertain pelvic masses; this case has a clear diagnosis.
D. Laparoscopy – Considered for suspected ovarian torsion, ruptured ectopic pregnancy, or severe hemorrhage; this patient is stable without signs of ongoing bleeding.
Question 26:
The correct answer is D. This patient presents with a 5-year history of intermittent dysphagia, regurgitation of undigested food, and aspiration pneumonia, which strongly suggests Zenker diverticulum.
Zenker diverticulum is a false diverticulum that forms at the pharyngoesophageal junction due to impaired cricopharyngeal muscle relaxation, leading to food retention, halitosis, and recurrent aspiration events.
Incorrect answers:
A. Achalasia – Causes progressive dysphagia to both solids and liquids, along with chest discomfort and weight loss, which are absent here.
B. Esophageal cancer – Presents with progressive dysphagia (first solids, then liquids), weight loss, and odynophagia, none of which are seen in this patient.
C. Hiatal hernia – Often presents with heartburn and regurgitation that improves with antacids, unlike the persistent regurgitation of undigested food seen in Zenker diverticulum.
Don’t let subpar interview performance prevent you from matching. Ace your residency interviews with our experts’ advice! Each session is half real-time mock interview and half as feedback.
Get Our Experts’ Guidance!
Question 27:
The correct answer is C. This patient is in active labor at 39 weeks’ gestation, showing regular contractions, progressive cervical dilation, and a reassuring fetal heart rate tracing (moderate variability, accelerations, no decelerations).
Labor is progressing normally, with cervical dilation from 6 cm to 9 cm within 30 minutes, indicating rapid progression. There are no maternal or fetal complications requiring intervention, so the best approach is expectant management, allowing labor to continue naturally.
Incorrect answers:
A. Advising the patient to begin pushing – Pushing is appropriate only when the cervix is fully dilated (10 cm). This patient is at 9 cm, so pushing prematurely could lead to maternal exhaustion or fetal distress.
B. Amnioinfusion – Indicated for variable decelerations due to umbilical cord compression, which are not present here.
D. Forceps-assisted vaginal delivery – Used when the cervix is fully dilated but delivery needs to be expedited due to maternal exhaustion or fetal distress. This patient is still in the active phase, making this intervention unnecessary.
E. Immediate cesarean delivery – Indicated for emergencies (e.g., cord prolapse, uterine rupture, persistent fetal distress). This patient has a reassuring fetal heart tracing and rapid cervical progress, so cesarean delivery is not required.
Question 28:
The correct answer is B. This patient presents with 3 months of postprandial epigastric pain without alarming features (e.g., weight loss, anemia, vomiting), which suggests peptic ulcer disease (PUD) or gastritis.
Endoscopy is the gold standard for diagnosing PUD and gastritis, allowing for direct visualization and biopsy to confirm the diagnosis and rule out H. pylori infection or malignancy.
Incorrect answers:
A. CT scan of the abdomen – More useful for detecting structural abnormalities (e.g., tumors, pancreatitis), not mucosal lesions like ulcers.
C. Examination of the stool for ova and parasites – Indicated for parasitic infections (e.g., Giardia, Entamoeba histolytica), which typically cause diarrhea and weight loss, not isolated epigastric pain.
D. Stool culture – Used for bacterial gastroenteritis (e.g., Salmonella, Shigella, Campylobacter), which presents with fever and diarrhea, not chronic postprandial pain.
E. Upper gastrointestinal series – Can detect ulcers, but is less sensitive than endoscopy and does not allow for biopsy or H. pylori testing.
Question 29:
The correct answer is C. This elderly woman with advanced dementia, dependent on a single caregiver, presents with multiple unexplained bruises and a distal radial fracture without a clear mechanism of injury. Her unkempt appearance and poor hygiene further suggest possible elder abuse and neglect.
Elder abuse should be considered in vulnerable populations, particularly when there are unexplained injuries, poor hygiene, and signs of neglect. Reporting to adult protective services is a legal and ethical obligation. In suspected elder abuse, mandatory reporting to Adult Protective Services (APS) is the appropriate next step to ensure the patient’s safety and initiate an investigation. Reporting to APC is an ethical obligation, even if you are not 100% sure. APS can assess the living environment and determine if further intervention, such as removal from the home or additional caregiver support, is necessary.
Incorrect answers:
A. Admit the patient to the hospital – Hospitalization provides temporary safety but does not address the underlying risk of ongoing abuse at home.
B & D. Arrange placement/provide phone numbers of nursing care facilities – Long-term placement may be needed, but reporting suspected abuse must occur first before determining next steps.
E. Recommend evaluation at a geriatric clinic – A comprehensive geriatric assessment may be beneficial, but it does not address the immediate safety concerns associated with potential abuse.
Question 30:
The correct answer is B. This child has poorly controlled asthma, evidenced by weekly nighttime cough and exercise-induced symptoms that limit activity. Despite not using albuterol frequently, these symptoms indicate the need for a controller medication. According to asthma treatment guidelines, the addition of an inhaled corticosteroid (ICS) is indicated for persistent asthma, as evidenced by symptoms.
Inhaled corticosteroids (ICS), such as fluticasone, are the preferred long-term controller therapy for persistent asthma. They reduce airway inflammation, preventing symptoms and exacerbations.
Incorrect answers:
A. Oral theophylline – An older asthma treatment with significant side effects (e.g., nausea, tachycardia, toxicity risks) and is not first-line therapy.
C. Salmeterol (LABA) – Long-acting beta-agonists should not be used alone due to the risk of severe exacerbations; they are always combined with ICS if needed.
D. Oral prednisone – Indicated for acute exacerbations, but this patient requires long-term control, not short-term symptom relief.
E. No change in management – The patient’s nighttime and activity-limiting symptoms confirm poorly controlled asthma, requiring treatment escalation.
Crush the Biostatistics of USMLE STEP 1, STEP 2 CK, and STEP 3 exams
Sign up NOW!
Question 31:
The correct answer is D. Premature closure occurs when a physician accepts a diagnosis too early without fully verifying it or considering alternative possibilities. Patients with complex medical conditions, such as cerebral palsy andsitus inversus, often have atypical presentations. Physicians must broaden their differential diagnosis and avoid anchoring bias to prevent missing critical conditions such as volvulus in this case.
In this case, the physician assumed gastroenteritis based on prior similar cases without considering the patient’s unique risk factors (cerebral palsy, situs inversus, severe scoliosis), which predispose her to volvulus. The failure to broaden the differential led to a missed diagnosis and delayed treatment, resulting in bowel necrosis and septic shock.
Incorrect answers:
A. Latent error – Refers to systemic or process-related failures (e.g., poor EHR design, understaffing) that increase the likelihood of errors, rather than a clinician’s cognitive bias.
B. Near miss – Describes an error that was caught before causing harm; in this case, the patient was harmed by the missed diagnosis.
C. Non-preventable error – This diagnosis could have been prevented with a more thorough evaluation.
E. Systems failure – Involves institutional breakdowns (e.g., equipment failure, poor communication between teams), whereas this error was due to individual clinical reasoning.
Question 32:
The correct answer is B. This patient presents with regional lymphadenopathy (tender axillary and epitrochlear nodes) and recent exposure to cats, which is highly suggestive of cat-scratch disease (CSD) caused by Bartonella henselae.
Key clinical features of CSD:
- History of cat scratches or bites, often from kittens.
- Localized lymphadenopathy (commonly axillary, cervical, or epitrochlear nodes).
- May have a small papule or pustule at the inoculation site.
- No systemic symptoms in immunocompetent individuals.
Incorrect answers:
A. Castleman disease – Presents with generalized lymphadenopathy, systemic symptoms (fever, night sweats), and organomegaly, which are not seen here.
C. Hidradenitis suppurativa – Chronic inflammatory skin disease with painful abscesses in the axilla, groin, and perineal areas; does not cause lymphadenopathy from a cat scratch.
D. T-cell lymphoma – Presents with widespread, persistent lymphadenopathy and systemic symptoms (fever, weight loss, night sweats); this patient’s localized lymphadenopathy and lack of systemic signs argue against lymphoma.
E. Tuberculosis – Causes chronic, painless lymphadenopathy, often with caseating necrosis and systemic symptoms; localized, tender lymph nodes after cat exposure are not typical.
Question 33:
The correct answer is A. This patient has clustered pleomorphic microcalcifications on mammography, which is a classic finding of ductal carcinoma in situ (DCIS), an early, noninvasive form of breast cancer.
Key radiologic features of DCIS:
- Microcalcifications that are pleomorphic, clustered, or linear.
- No palpable mass or skin changes (commonly detected on routine mammography).
- Considered a precancerous lesion with a risk of progression to invasive ductal carcinoma.
Incorrect answers:
B. Fat necrosis – Associated with trauma or prior breast surgery, presents as a palpable mass, often with oil cysts and coarse calcifications, rather than clustered pleomorphic microcalcifications.
C. Fibroadenoma – Presents as a well-defined, mobile solid mass, not microcalcifications.
D. Mastitis – Causes breast redness, warmth, tenderness, and systemic symptoms (fever, malaise), which are absent here.
E. Sclerosing adenosis – A benign proliferative breast condition that can have calcifications, but they are not pleomorphic or clustered.
Question 34:
The correct answer is F. This patient, after a head-on motor vehicle collision, presents with decreased breath sounds on the right, chest and abdominal pain, and hypoxia (PaO₂ of 65 mm Hg) on ABG analysis, all of which are highly suggestive of hemothorax. Hemothorax occurs when blood accumulates in the pleural space due to trauma, and the initial management for this is tube thoracostomy (chest tube placement).
A tube thoracostomy allows for drainage of the blood and air from the pleural space, improving lung expansion and oxygenation. This intervention is crucial in preventing respiratory distress and complications from continued bleeding or fluid accumulation.
Incorrect answers:
A. Bronchoscopy – Typically used for airway management, foreign body removal, or visualization of airway pathology. Not indicated for traumatic hemothorax.
B. Intubation and mechanical ventilation – Indicated for respiratory failure. However, this patient’s hypoxia can be managed with a tube thoracostomy and supplemental oxygen, not necessarily mechanical ventilation at this stage.
C. Placement of a thoracic epidural – Typically used for pain control in thoracic surgeries or chronic pain management, not for trauma-related conditions like hemothorax.
D. Thoracentesis – Primarily used for diagnosing and draining pleural effusions; not for hemothorax. Tube thoracostomy is needed to drain significant blood accumulation in the pleural space.
E. Thoracotomy – A more invasive procedure, typically reserved for massive hemothorax or non-responsive cases; not the first-line treatment.
Feeling swamped by the complexities of Systematic Reviews? Kickstart your research journey with our comprehensive Systematic Review Course and build your skills with confidence!
Sign up NOW!
Question 35:
The correct answer is E. This patient initially had proteinuria during a urinary tract infection (UTI), which has since resolved on repeat urinalysis. Transient proteinuria can occur due to fever, infection, exercise, or stress, and it does not require further workup if it resolves.
Since the follow-up urinalysis is completely normal, no additional testing is necessary.
Incorrect answers:
A. Determination of urine albumin:creatinine ratio – Used for chronic proteinuria, such as in diabetic nephropathy. Not needed here because follow-up urinalysis is normal.
B. 24-hour urine collection for protein concentration – Used to quantify proteinuria in chronic kidney disease; unnecessary since proteinuria has resolved.
C. Ultrasonography of the kidneys – Indicated for structural abnormalities or renal obstruction. There is no suspicion of kidney disease in this patient.
D. Urine sulfosalicylic acid method – Used to detect non-albumin proteins (e.g., Bence-Jones proteins in multiple myeloma). This patient does not have persistent proteinuria or signs of multiple myeloma.
Question 36:
The correct answer is D. This patient presents with a 2-week history of sore throat, anterior neck tenderness/fullness, fever, tachycardia, and no oropharyngeal exudates, which is highly suggestive of subacute thyroiditis (De Quervain’s thyroiditis).
Subacute thyroiditis is a self-limited, inflammatory disorder of the thyroid gland that often follows a viral upper respiratory infection. It leads to painful thyroid swelling, transient thyrotoxicosis, and systemic symptoms (e.g., fever, tachycardia, anxiety).
Incorrect answers:
A. Acute mononucleosis – Typically presents with fever, pharyngitis, posterior cervical lymphadenopathy, and atypical lymphocytosis; this patient lacks lymphadenopathy and pharyngeal exudates.
B. Gastroesophageal reflux disease (GERD) – Can cause chronic sore throat, but it does not cause fever, neck tenderness, or systemic symptoms.
C. Laryngitis – Characterized by hoarseness and voice changes; does not cause neck tenderness or thyroid involvement.
E. Tracheitis – Often caused by bacterial infections, presents with high fever, cough, and airway obstruction, and does not involve thyroid tenderness.
Do You Want to be Part of Our Match Success Stories?
Explore The Match Packages!
Trustpilot
Trustpilot
Question 37:
The correct answer is A. This patient meets the criteria for generalized anxiety disorder (GAD), evidenced by excessive worry about multiple domains (finances, job) for at least six months, along with associated symptoms of sleep disturbance, muscle tension, irritability, and fatigue.
In patients with a history of substance use disorder, buspirone is a preferred second-line non-addictive anxiolytic. While cognitive-behavioral therapy (CBT) and SSRIs (e.g., sertraline, escitalopram) are first-line treatments, buspirone is an alternative when SSRIs are not tolerated or preferred.
Incorrect answers:
B. Clonazepam is a benzodiazepine, which can cause dependence and relapse in patients with a history of substance use.
C. Diphenhydramine is an antihistamine, which can cause sedation but does not treat the underlying anxiety.
D. Imipramine is a tricyclic antidepressant (TCA), which is not first-line due to side effects (anticholinergic, cardiotoxicity). SSRIs are preferred for GAD.
E. Quetiapine is an atypical antipsychotic, which is not indicated for primary anxiety disorders due to metabolic side effects.
F. Temazepam is a benzodiazepine, inappropriate for this patient due to risk of dependence and addiction relapse.
Question 38:
The correct answer is C. This patient presents with acute respiratory distress, hypotension, and a widespread erythematous rash following amoxicillin exposure, which is highly suggestive of anaphylaxis.
Key diagnostic features of anaphylaxis:
- Cutaneous symptoms (e.g., erythematous, raised, demarcated rash)
- Respiratory distress (e.g., increased respiratory rate, dyspnea)
- Cardiovascular involvement (e.g., hypotension)
- Rapid onset after allergen exposure (e.g., amoxicillin use)
Epinephrine (intramuscular) is the first-line treatment for anaphylaxis because it:
- Reverses airway obstruction (bronchodilation)
- Increases blood pressure (vasoconstriction)
- Reduces swelling (decreasing capillary permeability)
Incorrect answers:
A. Administration of albuterol – Used for bronchospasm (e.g., asthma, COPD) but does not treat the systemic effects of anaphylaxis.
B. Administration of diphenhydramine – An H1 antihistamine, helpful as adjunctive therapy, but not first-line in anaphylaxis.
D. Complete blood count – Does not address the acute, life-threatening situation; immediate treatment is needed.
E. Observation only – Anaphylaxis is a medical emergency requiring immediate epinephrine to prevent airway compromise and cardiovascular collapse.
Unlock your research potential with our dynamic course designed for medical students and graduates! Built by an experienced clinical researcher with 150+ publications and 3000+ citations!
Sign up NOW!
Question 39:
The correct answer is D. This patient’s chronic hematochezia, iron deficiency anemia, loose stools, and colonoscopy findings of coalesced ulcers with normal intervening mucosa (skip lesions) involving the ileum are highly suggestive of Crohn’s disease.
Key points about Crohn’s disease:
- Chronic inflammatory bowel disease affecting any part of the GI tract (commonly the terminal ileum and colon).
- Skip lesions with deep ulcers and normal intervening mucosa (vs. continuous inflammation in ulcerative colitis).
- Can cause anemia (due to malabsorption and chronic blood loss).
Smoking is a major risk factor for Crohn’s disease and is associated with:
- More aggressive disease progression
- Increased risk of complications (e.g., strictures, fistulas, surgery)
- Poor response to medical therapy
Smoking cessation is strongly recommended in all Crohn’s patients as it improves outcomes and reduces disease severity.
Incorrect answers:
A. Annual screening for lymphoproliferative disorders – Not routinely recommended in Crohn’s disease unless the patient is on immunosuppressive therapy (e.g., azathioprine, methotrexate).
B. Discontinuation of citalopram therapy – SSRIs (e.g., citalopram) do not worsen Crohn’s disease; anxiety is not a contraindication for treatment.
C. Prophylactic colectomy – Unlike ulcerative colitis, Crohn’s disease is not curable with colectomy since it can affect the entire GI tract. Surgery is reserved for complications (e.g., strictures, fistulas, perforation).
Question 40:
The correct answer is A. This patient has psychogenic non-epileptic seizures (PNES), as evidenced by:
- Seizure-like episodes without EEG changes during long-term video EEG monitoring.
- Prolonged duration (15-20 minutes), bilateral limb shaking, head movements, crying, and intermittent responsiveness, which are atypical for epileptic seizures.
- History of childhood trauma (sexual and physical abuse), a known risk factor for PNES.
- Lack of response to multiple antiepileptic drugs (AEDs), suggesting a non-epileptic cause.
PNES is a functional neurological disorder (conversion disorder) that manifests as seizure-like episodes without epileptiform activity. It is a psychological condition, not a true seizure disorder.
Cognitive Behavioral Therapy (CBT) is the first-line treatment for PNES, as it helps patients understand and manage psychological distress, reducing symptom frequency.
Incorrect answers:
B. Hypnotic therapy – Not the primary treatment for PNES; lacks strong evidence compared to CBT.
C. Increasing the dosage of pregabalin – PNES is not epilepsy, so additional AEDs provide no benefit and may cause unnecessary side effects.
D. Psychoanalytic therapy – CBT is the evidence-based approach, while psychoanalysis is less effective for PNES.
E. Surgical resection of the epileptogenic focus – There is no epileptogenic focus in PNES; surgery is only considered for medically refractory focal epilepsy with a clear seizure focus.
Ace Your Step 2 CK with Expert Guidance
Struggling with Step 2 CK preparation? Get personalized 1-on-1 coaching from top scorers and experienced mentors.
Book a Session Today!
Trustpilot
Trustpilot
Block 2
Question 41:
The correct answer is A. Ask clinical providers in the unit to identify what they believe to be the most common factors leading to non-removal of central venous catheters.
Central venous catheters increase the risk of bloodstream infections when left in place for prolonged periods. The first step in addressing this issue is identifying the root causes. Engaging clinical providers directly involved in patient care ensures a better understanding of barriers to timely catheter removal and helps develop effective, real-world strategies. On the USMLE, quality improvement questions often emphasize prioritizing root cause analysis before implementing interventions.
Incorrect answers:
B. Changing all central venous catheters every five days: This approach leads to unnecessary procedures and does not address the underlying reasons for prolonged catheter use.
C. Placing stickers: A superficial solution that does not target specific barriers to catheter removal.
D. Randomized controlled trial: Although valuable for research, it is time-consuming and inefficient as an initial step in a quality improvement process.
Question 42:
The correct answer is C. Administration of Rh(D) immune globulin
Rho(D) immune globulin is administered prophylactically to prevent Rh-negative mothers from becoming sensitized to Rh-positive fetal red blood cells, which can occur after events like vaginal bleeding. Sensitization risks fetal hemolytic disease in future pregnancies. For the USMLE/NBME, recognize that Rho(D) immune globulin is crucial after any potential fetomaternal hemorrhage, even if anti-D antibody titers are weakly positive.
Incorrect answers:
A. Repeat serum anti-D antibody titer: Does not prevent sensitization or protect against complications.
B. Ultrasonography of the pelvis: Does not address the risk of Rh sensitization or influence management.
D. Amniocentesis: Invasive and not indicated solely for assessing Rh sensitization.
E. Induction of labor: Premature delivery at 28 weeks is unwarranted without a compelling reason, and Rh sensitization alone does not justify it.
Question 43:
The correct answer is C. Kaposi sarcoma
Kaposi sarcoma (KS) typically presents as painless, violaceous patchy (as in this scenario), macular or nodular lesions, often seen in patients with underlying immunosuppression, such as HIV/AIDS. The systemic symptoms, including weight loss in this patient, raise suspicion for an associated neoplasm or systemic disease. KS is linked to Human Herpesvirus 8 (HHV-8) and is a dermatologic malignancy that should be high on the differential in such cases. On the USMLE, a violaceous rash with systemic symptoms is a classic clue for Kaposi sarcoma.
Incorrect answers:
A. Actinic keratosis: Appears as rough, scaly lesions on sun-exposed areas, typically precancerous, without systemic symptoms like weight loss.
B. Dermatitis herpetiformis: Characterized by intensely pruritic, grouped vesicles associated with celiac disease, not violaceous lesions or weight loss.
D. Lichen planus: Presents with polygonal, purple, pruritic papules and plaques, often with Wickham’s striae, but not systemic symptoms.
E. Seborrheic dermatitis: Features scaly, erythematous patches, commonly affecting the scalp and face, without weight loss or systemic involvement. Severe and Widespread dermatitis is associated with HIV
Our Clients’ Success Speaks to Our Premium Service!
The Match Application Packages
Question 44:
The correct answer is B.Elder neglect
This patient’s worsening self-care, including missed appointments, unkempt appearance, and deteriorating diabetes management, coincides with her son moving in, which raises concerns for elder neglect. Elder neglect often manifests as a failure to meet basic needs, such as medical care, hygiene, and nutrition, without signs of overt physical abuse. Recognizing these patterns is critical for USMLE questions, where socio-environmental factors may explain a sudden decline in a previously stable elderly patient.
Incorrect answers:
A. Dietary indiscretion: While poor dietary choices could elevate glucose, they would not account for missed appointments or unkempt appearance.
C. Major depressive disorder: Although the patient has a flat affect, she lacks other hallmark symptoms such as anhedonia, pervasive sadness, or fatigue.
D. Pancreatic malignancy: The patient has no weight loss, abdominal pain, or systemic symptoms commonly seen in pancreatic cancer.
E. Urinary tract infection: The urinalysis does not show findings typical of a UTI, such as nitrites, leukocyte esterase, or pyuria.
Question 45:
The correct answer is A. Inform the son that the decision is the patient’s to make, regardless of the medical team’s recommendation.
The patient demonstrates intact decision-making capacity, as he is awake, alert, and able to converse. In this context, patient autonomy takes precedence, and his healthcare decisions must be respected. A power of attorney (POA) is only activated when a patient cannot make their own decisions. It is essential to understand that a competent patient’s rights override any recommendations or preferences from family members or healthcare providers.
Incorrect answers:
B. Follow the physical therapist’s recommendation: The patient’s decision takes precedence over any healthcare provider’s recommendation if he has the capacity to decide.
C. Arrange a family meeting: While family meetings can help in some cases, they are unnecessary here because the patient’s decision is clear and should be honored.
D. Obtain a psychiatry consultation: There is no evidence that the patient lacks mental competency; he is fully conversant and aware of his situation.
E. Review the power of attorney: A POA does not override the decisions of a competent patient, so this step is unnecessary.
Question 46:
The correct answer is A. Abstinence from alcoholic beverages
Alcohol use is a leading cause of acute pancreatitis, and continued alcohol consumption greatly increases the risk of recurrent episodes. By abstaining from alcohol, the patient addresses the primary etiological factor of her condition, making it the most effective preventive measure. On the USMLE, always prioritize interventions that directly target the root cause of a condition, especially when the vignette highlights a modifiable risk factor like alcohol use in this case.
Incorrect answers:
B. Enrollment in a health insurance plan: While important for access to healthcare, it does not directly address the primary cause of her pancreatitis.
C. Nutritional counseling: Although diet modifications can benefit pancreatitis patients, alcohol is the key modifiable risk factor in this scenario.
D. Placement in a women’s shelter: This might help in addressing social issues but does not directly mitigate the risk of alcohol-induced pancreatitis unless it involves substance abuse intervention.
Our Clients’ Success Speaks to Our Premium Service!
Our Interview Preparation Packages
Question 47:
The correct answer is E.Mirtazapine
Mirtazapine is an atypical antidepressant that effectively treats depression and is particularly useful in patients with insomnia due to its sedative properties. For patients with epilepsy, it is also a safer option because it does not lower the seizure threshold, unlike some other antidepressants. In this case, the patient’s symptoms of depressed mood, fatigue, insomnia, and decreased interest are consistent with major depressive disorder, and mirtazapine is a well-suited option given the coexisting sleep disturbances and epilepsy.
Incorrect answers:
A. Alprazolam: A benzodiazepine used primarily for anxiety disorders, not depression, and carries a risk of dependence.
B. Aripiprazole: An antipsychotic used as an adjunct in treatment-resistant depression, but not a first-line option for this patient’s presentation.
C. Bupropion: While effective for depression, it is activating and lowers the seizure threshold, making it unsuitable for patients with epilepsy.
D. Buspirone: Used for generalized anxiety disorder, it is not an antidepressant and would not address this patient’s depressive symptoms.
Question 48:
The correct answer is B. Serum albumin concentration
The patient’s periorbital and leg edema, along with proteinuria, suggests nephrotic syndrome, which is characterized by hypoalbuminemia. Low serum albumin leads to reduced oncotic pressure, causing fluid to accumulate in tissues and resulting in edema.
Minimal change disease, the most common cause of nephrotic syndrome in children, typically presents with these symptoms without significant hematuria or inflammation. In this case, testing serum albumin will be an additional finding.
Incorrect answers:
A. Hemoglobin: Anemia is not typically associated with nephrotic syndrome.
C. Serum C3 and C4 concentrations: Decreased complement levels are more likely to be seen in nephritic syndromes like PSGN.
D. Serum triglycerides concentration: Triglycerides are usually elevated, not decreased, in nephrotic syndrome.
E. Serum urea nitrogen concentration: Urea levels may be normal or slightly elevated but don’t explain the edema.
Question 49:
The correct answer is B. Contact the organ bank for potential donation.
This patient’s presentation suggests brain death, with signs such as loss of consciousness, loss of brainstem reflexes (absent corneal reflex, absent eye movements), and a significant brain injury likely due to a ruptured cerebral aneurysm. If brain death is suspected, the patient may be eligible for organ donation. The absence of an advance directive does not prevent organ donation if the family or surrogate decision-makers are aware of the patient’s wishes.
Incorrect answers:
A. Consult with the hospital ethics committee: This can be a later step, but the immediate priority is contacting the organ bank.
C. Determine if patients are awaiting organ donation: Organ allocation is handled by regional organ procurement organizations, not individual hospitals.
D. Lack of advance directive: Does not prevent organ donation, especially with family consent.
E. History of hypertension: Hypertension is not a contraindication for organ donation
Would You Like Us to Help You Ace Your STEP Exams?
Learn More!
Trustpilot
Trustpilot
Question 50:
The correct answer is A. Corticosteroid injection.
Keloids are characterized by an overproduction of collagen during wound healing, often occurring after trauma such as ear piercings. The most effective treatment for smaller keloids is intralesional corticosteroid injection, which reduces the size and itchiness of the keloid by inhibiting collagen synthesis and enhancing collagenase activity.
Incorrect answers:
B. Cryotherapy: Typically used for warts or actinic keratoses, not for keloids.
C. Laser ablation: Reserved for refractory keloids and not first-line treatment.
D. Needle aspiration: Keloids are solid and fibrous, making aspiration ineffective.
E. Wide excision: Surgery can exacerbate keloids, making it less ideal as an initial treatment.
Question 51:
The correct answer is D. Ruptured abdominal aortic aneurysm (AAA).
The patient’s sudden onset of severe abdominal pain, hypotension, and a history of hypertension and smoking strongly point toward a ruptured AAA. AAA rupture commonly presents with abdominal or back pain, hypotension, and signs of hemorrhagic shock. Smoking is the most significant modifiable risk factor for developing AAA.
Incorrect answers:
A. Acute pancreatitis: While it causes severe abdominal pain, it’s typically epigastric, radiating to the back, and associated with elevated serum amylase or lipase. Hypotension is uncommon unless complicated by severe systemic inflammation.
B. Mesenteric ischemia: Can cause severe abdominal pain, but hypotension is not the primary presentation. Pain is often “out of proportion” to physical findings, and patients have risk factors like atrial fibrillation or atherosclerosis.
C. Perforated duodenal ulcer: May cause sudden epigastric pain and signs of peritonitis (e.g., rigid abdomen). Abdominal X-ray typically shows air under the diaphragm.
E. Small bowel obstruction: Typically presents with crampy abdominal pain, vomiting, and distension, but does not cause sudden hypotension or hemorrhagic shock.
Question 52:
The correct answer is C. DASH diet
The patient’s blood pressure is consistent with stage 1 hypertension (<140/90 mmHg). In cases without significant cardiovascular risk factors or end-organ damage, lifestyle interventions such as the DASH diet, weight loss, and exercise are the first-line approach. The DASH diet is particularly effective, emphasizing fruits, vegetables, whole grains, and low-fat dairy, which have been shown to reduce blood pressure significantly.
Incorrect answers:
A. ACE inhibitor therapy: Antihypertensive medications are typically reserved for stage 2 hypertension or stage 1 hypertension with evidence of end-organ damage or additional cardiovascular risk factors.
B. Calcium supplementation: The DASH diet naturally provides adequate calcium and has a more established role in reducing blood pressure.
D. Fish oil supplementation: While omega-3 fatty acids have cardiovascular benefits, they do not specifically target blood pressure reduction.
E. Thiazide diuretic therapy: Like ACE inhibitors, medications are reserved for more severe hypertension or when lifestyle modifications are insufficient.
Become a Researcher in the United States
No Experience Required! Just join the live & interactive workshop to learn all the secrets of landing a research position in the U.S
Enroll Here
Trustpilot
Trustpilot
Question 53:
The correct answer is A. Administer a bolus of intravenous 3% saline
The patient’s generalized tonic-clonic seizure combined with severe hyponatremia (Na <125 mEq/L) necessitates immediate treatment with hypertonic saline (3% saline). This step is critical to increase serum sodium levels and prevent further neurological complications, including brain herniation due to cerebral edema. However, caution should be taken while administering a bolus of 3% saline as rapid correction could lead to Central Pontine Myelinolysis (CPM). (Mnemonic- Low to high your pons will die; High to low your brain will blow)
Incorrect answers:
B. Subcutaneous vasopressin: Usually used for central diabetes insipidus, not for correcting acute, severe hyponatremia.
C. Initiate fosphenytoin therapy: Although seizures occurred, the root cause is hyponatremia, not epilepsy. Treating the underlying issue is the priority.
D. 0.9% saline at 1.5 times maintenance: Isotonic saline raises sodium levels too slowly in symptomatic, severe hyponatremia.
E. Order EEG: Not immediately necessary since the cause of the seizure (hyponatremia) is evident.
F. Order MRI of the brain: Imaging is unwarranted here as the symptoms clearly stem from the electrolyte abnormality.
Question 54:
The correct answer is A. Albuterol therapy
The patient’s sudden onset of coughing spasms, wheezing, and shortness of breath after exposure to cleaning agents strongly suggests chemical-induced bronchospasm. Albuterol, a short-acting beta-2 agonist, provides rapid bronchodilation and symptom relief, making it the first-line treatment for this acute condition.
Incorrect answers:
B. Bronchoscopy: Not indicated as a first-line intervention. It is typically used for airway evaluation or foreign body removal but not for acute bronchospasm.
C. CT scan of the chest: Not immediately necessary. Imaging is reserved for cases with unclear diagnosis or complications.
D. Intubation: The patient is not in respiratory failure or severe distress, so invasive airway management is unnecessary.
E. Methylprednisolone therapy: Steroids may help reduce inflammation in asthma or severe bronchospasm but are not the initial treatment for acute symptoms.
Question 55:
The correct answer is D. Measurement of urine and plasma osmolarity
The patient’s confusion, low serum sodium, and history of schizophrenia suggest hyponatremia, which requires further evaluation to determine the underlying cause. Measuring urine and plasma osmolarity is the next step to classify the hyponatremia as hypo-, iso-, or hyperosmolar, guiding appropriate further management.
Incorrect answers:
A. CT scan of the chest: While lung pathology (e.g., small cell lung cancer causing SIADH) can lead to hyponatremia, this is not the first step. Focus on osmolarity studies first.
B. Measurement of serum cortisol and TSH concentrations: Although adrenal insufficiency or hypothyroidism can cause hyponatremia, these are less likely without other suggestive signs (e.g., hyperpigmentation or bradycardia).
C. Measurement of serum uric acid concentration: Uric acid may decrease in SIADH but is not the primary diagnostic tool in hyponatremia evaluation.
E. MRI of the brain: Neurologic symptoms are more likely secondary to hyponatremia. Imaging is reserved for cases where symptoms persist after addressing the electrolyte disturbance.
Get access to our list of Electives and U.S. Clinical Experience for IMGs!
Learn More
Question 56:
The correct answer is C. Coronary artery disease
The patient’s long-standing type 1 diabetes mellitus and ST-segment depression on the exercise stress test indicate myocardial ischemia, pointing to coronary artery disease (CAD). Diabetes, especially when uncontrolled over many years, significantly increases the risk of atherosclerosis and CAD. The ST-segment depression suggests ischemia, making CAD the most likely diagnosis. In a diabetic patient, even in the absence of chest pain (due to potential diabetic neuropathy causing silent myocardial ischemia), significant ST-segment changes on a stress test strongly point towards CAD.
Incorrect answers:
A. Cardiomyopathy: No signs of heart failure are present, and the murmur doesn’t fit the typical pattern of cardiomyopathy.
B. Congenital heart disease: The murmur and the patient’s age make this diagnosis unlikely.
D. Valvular heart disease: While murmurs are common, valvular heart disease wouldn’t explain the ST-segment changes seen in the stress test.
E. Normal cardiac findings: The ST-segment depression is abnormal and suggests ischemia, not a normal finding.
Question 57:
The correct answer is B. Diagnosis-driven reminders in patient charts
Diagnosis-driven reminders in patient charts are effective because they directly prompt healthcare providers about the required care steps for specific conditions. These reminders can be automated within the electronic medical record system and ensure that important care measures, like pulmonary function testing for asthma, are not overlooked.
Incorrect answers:
A. Annual chart reviews and feedback: These can be time-consuming and may not provide immediate feedback to influence care at the time of patient visits.
C. Flyers in the waiting room: They are not provider-directed and may not be sufficient to prompt action from either the provider or the patient.
D. Testing all patients with respiratory symptoms: This is inefficient and not specific to asthma, leading to over-testing.
E. Testing post-exacerbation: This is too specific and misses the need for regular testing in stable asthma patients.
Question 58:
The correct answer is E. Parkinson’s disease
The patient’s symptoms are characteristic of REM sleep behavior disorder (RBD), which involves acting out vivid dreams, often violently, during REM sleep. RBD is strongly associated with neurodegenerative diseases, particularly Parkinson’s disease. Additionally, the patient’s history of constipation, a common non-motor symptom, supports the diagnosis of Parkinson’s disease, which often presents with both motor and non-motor symptoms.
Incorrect answers:
A. Alzheimer’s disease: Although Alzheimer’s disease can lead to cognitive and behavioral changes, RBD is more strongly linked to Parkinson’s disease than to Alzheimer’s.
B. Amyotrophic lateral sclerosis: ALS does not typically present with RBD, and it is not associated with the motor and sleep disturbances described here.
C. Cerebral infarction: There is no direct link between cerebral infarction and RBD symptoms.
D. Narcolepsy: Narcolepsy typically presents with excessive daytime sleepiness, cataplexy, sleep paralysis, and hypnagogic hallucinations but not with violent dream enactment.
What Do Our MATCH Application Packages Include?
Advisor UNLIMITED Access
We get how stressful the residency match process is, so we're here for you - communicate with your personal advisor ANYTIME you need!
Personal Statement Editing
Our editing includes not only language but also context, structure, and content advising.
ERAS Application Editing
The editing goes beyond language and grammar corrections to structure, design, and content based on your personal story and achievement.
Interview Preparation
The best way to learn something is to do it. That’s why we divide our interview preparation sessions into two parts.
Mock Interview + Feedback
Sign up NOW!
Question 59:
The correct answer is A. Agoraphobia
This patient is experiencing panic attacks, characterized by sudden onset of intense fear and somatic symptoms like rapid heartbeat, sweating, and nausea. The patient also describes avoiding situations due to the fear of having another attack, which is a hallmark of agoraphobia. For USMLE/NBME, understanding that agoraphobia arises when patients avoid places or situations where they might have a panic attack is key.
Incorrect answers:
B. Generalized anxiety disorder: This disorder involves chronic, excessive worry about multiple aspects of life, rather than specific, situational panic attacks.
C. Illness anxiety disorder: The patient is not excessively worried about having a serious illness but rather about the occurrence of panic attacks.
D. Social anxiety disorder: The fear here is of social situations where one might be scrutinized by others, not fear of panic attacks in specific situations.
E. Somatic symptom disorder: This condition involves physical symptoms causing distress, but it is not linked to the intense fear and avoidance behavior seen in agoraphobia.
Question 60:
The correct answer is E. Transabdominal ultrasonography
In cases of late-pregnancy vaginal bleeding, placenta previa (PP) must be considered. PP occurs when the placenta partially or completely covers the cervix, which can lead to painless, bright red bleeding in the third trimester. Transabdominal ultrasonography is the first-line diagnostic tool to evaluate the placenta’s location and rule out or confirm placenta previa. This non-invasive approach is essential to prevent further complications like hemorrhage, especially before performing any vaginal examination (Contraindicated in PP).
Incorrect answers:
A. Cesarean delivery now: A cesarean delivery should not be done without a confirmed diagnosis. It’s premature to decide on surgical delivery before identifying the underlying cause of the bleeding.
B. Digital examination of the cervix: A digital exam should be avoided in cases of unexplained third-trimester bleeding due to the risk of stimulating hemorrhage in the presence of placenta previa.
C. Magnesium sulfate: Typically used for preeclampsia or preterm labor, magnesium sulfate does not address the primary concern of vaginal bleeding due to placenta previa.
D. Oxytocin: Oxytocin is used to induce labor, but it is not indicated in the case of unexplained vaginal bleeding, especially without a confirmed diagnosis of labor.
Question 61:
The correct answer is C. X-rays of the left hand and wrist
The girl’s short stature and delayed puberty point to a possible delay in skeletal maturation. X-rays of the left hand and wrist, known as a bone age study, are used to assess skeletal maturity and help determine future growth potential. A delayed bone age compared to chronological age suggests that the patient may have constitutional growth delay, which is a normal variant where growth is delayed but ultimately follows a typical growth trajectory once puberty begins.
Incorrect answers:
A. Complete blood count and serum chemistry panel: These tests would be more helpful if there were signs of anemia, infection, or metabolic disturbances. However, they are not the first step in assessing growth delay without other concerning symptoms.
B. Measurement of urine-free cortisol concentration: This test is relevant in evaluating Cushing’s syndrome, but there is no evidence in the history or physical exam to suggest this diagnosis.
D. X-rays of the long bones and spine: These are more useful for evaluating skeletal dysplasias or bony pathologies, but in this case, the focus is on assessing the girl’s growth potential, which is best done with a bone age study.
E. X-ray of the skull: This would be relevant if there were concerns about cranial abnormalities (e.g., craniosynostosis), but there is no indication of such a problem here.
Boost your USMLE scores with personalized, one-on-one tutoring from our expert tutors at The Match Guy!
Learn More!
Trustpilot
Trustpilot
Question 62:
The correct answer is C. Hepatocellular carcinoma
The patient’s history of mild to moderate fatigue, elevated liver enzymes, and chronic hepatitis B infection (indicated by HBsAg positive and IgG anti-HBc positive) raises concern for hepatocellular carcinoma (HCC). Chronic hepatitis B is a well-established risk factor for HCC, and these subtle symptoms can precede the development of cancer.
Incorrect answers:
A. Amyloidosis: Although amyloidosis can affect multiple organs, it is not typically associated with chronic hepatitis B infection and does not fit the clinical picture here.
B. Essential mixed cryoglobulinemia: This condition is more strongly associated with hepatitis C, not hepatitis B. It is characterized by immune complex deposition and is not a common complication of hepatitis B.
D. Membranoproliferative glomerulonephritis: It’s more common with hepatitis C than B. Overall, it is much less likely.
E. Polyarteritis nodosa: Although polyarteritis nodosa can be associated with hepatitis B, it is less likely compared to HCC.
F. Sjögren syndrome: This autoimmune condition primarily affects exocrine glands (such as salivary and lacrimal glands) and does not have a strong association with hepatitis B or hepatocellular carcinoma.
Question 63:
The correct answer is H. Pulmonary infarction
The patient’s symptoms of pleuritic chest pain, hemoptysis, recent surgery, and the radiographic finding of a pleural-based triangular density (Hampton’s hump) strongly suggest pulmonary infarction. This condition occurs due to a blockage in the pulmonary vasculature, commonly caused by pulmonary embolism (PE), which can be difficult to detect if no obvious DVT is present. The post-surgical setting increases the risk for thromboembolic events, making pulmonary infarction a likely diagnosis.
Incorrect answers:
A. Air embolism: Typically occurs immediately after surgical or invasive procedures, not days later. It also tends to cause cardiovascular collapse, which is not observed here.
B. Empyema: Empyema typically presents with fever and purulent sputum, not blood-streaked sputum. The patient’s lack of fever makes this diagnosis less likely.
C. Hemothorax: This would likely present with massive hemoptysis and shock, not the localized pain described here. The x-ray would also typically show a large collection of blood in the pleural space, which is not consistent with the current presentation.
D. Pericarditis: The x-ray findings do not support pericarditis, and there is no mention of a pericardial friction rub, which would be a key feature in pericarditis.
E. Pneumonia: Pneumonia usually presents with fever, productive cough, and systemic symptoms, which are absent in this patient.
F. Pulmonary hemorrhage: Pulmonary hemorrhage typically results from severe inflammation or trauma, and there is no history to support this in the patient’s case.
G. Pulmonary hypertension: This is a chronic condition and would not cause acute chest pain in this scenario.
Question 64:
The correct answer is D. Spinal stenosis
The patient’s history of back pain radiating into both legs, which is worsened by standing or walking and relieved by sitting or flexing the spine, is characteristic of neurogenic claudication due to lumbar spinal stenosis. The symptoms typically arise from pressure on the spinal nerves in the lumbar region, and degenerative changes with age are a common cause. It’s important to recognize that spinal stenosis pain is positional and worsens with lumbar extension (standing or walking), whereas relief is obtained with lumbar flexion (sitting or leaning forward).
Incorrect answers:
A. Abdominal aortic aneurysm: Typically asymptomatic unless ruptured and would not explain the positional nature of leg pain and tingling seen in this patient.
B. Osteoporosis: While it can cause back pain, osteoporosis would not cause radicular pain or tingling in the legs, and it does not have the positional characteristics of spinal stenosis.
C. Peripheral neuropathy: Causes distal symmetric numbness or pain, typically not associated with positional changes. It also lacks the relief with flexion and worsening with extension seen in spinal stenosis.
E. Vascular claudication: Characterized by leg pain that is relieved by rest, not positional changes like lumbar flexion. Additionally, vascular claudication does not typically cause tingling, and the ankle-brachial index would be abnormal in cases of peripheral arterial disease.
Need Help Strengthening Your Research Experience?
Residency programs love applicants with strong research experience! Learn how to publish research, get involved in projects, and impress program directors with our comprehensive clinical research course.
Learn More!
Trustpilot
Trustpilot
Question 65:
The correct answer is C. Fasting glucose tolerance test
The patient’s history of delivering large-for-gestational-age infants, her obesity (BMI of 38 kg/m²), and a family history of type 2 diabetes place her at increased risk for gestational diabetes mellitus (GDM). In such cases, GDM should be screened at the first prenatal visit. The screening usually involves an initial glucose challenge, followed by a more definitive glucose tolerance test if the challenge test is abnormal. For USMLE/NBME, recognize that early screening is essential for high-risk patients like this one.
Incorrect answers:
A. Antiphospholipid antibody screening: This test is typically performed when there is a history of recurrent miscarriages or stillbirths, but it is not indicated based solely on this patient’s history of large babies.
C. Chest x-ray: There is no indication of a respiratory or thoracic issue in this patient, so this test is unnecessary.
D. Measurement of serum α-fetoprotein: This is used for neural tube defect screening, typically between 15-20 weeks’ gestation, not as a general screening in this case.
E. Measurement of serum-free thyroxine: This test would be relevant if there were concerns about thyroid dysfunction, but no such clinical evidence is presented here.
Question 66:
The correct answer is C. Excessive alcohol use
The patient’s daily consumption of six beers, which equates to more than two drinks per day, indicates excessive alcohol use—a well-known reversible cause of hypertension. Chronic alcohol intake, defined as more than two drinks per day for men, is a significant modifiable risk factor for elevated blood pressure. In young adults, alcohol use can contribute to sustained hypertension, and reducing alcohol intake can often normalize blood pressure.
Incorrect answers:
A. Cigarette smoking: Although smoking contributes to cardiovascular risk, it is not typically associated with chronic hypertension. Smoking is more linked with atherosclerosis and other heart diseases but not as directly with sustained hypertension.
B. Essential hypertension: Essential hypertension is common, but it is a diagnosis of exclusion. In young adults, it’s important to first rule out secondary causes, like excessive alcohol use, before diagnosing primary hypertension.
D. Pheochromocytoma: This condition often presents with episodic symptoms such as hypertension, headache, sweating, and tachycardia. The patient’s daily, persistent hypertension makes pheochromocytoma less likely.
E. Renal artery stenosis: While renal artery stenosis can cause hypertension in young adults, it typically presents with additional findings, such as a bruit over the abdomen or flank pain. In this case, alcohol use is a more likely contributor to the patient’s hypertension.
Question 67:
The correct answer is A. Areas of eschar at the center of the wound site
The patient’s presentation, which includes a red, severely painful lesion after what is likely a spider bite, suggests a brown recluse spider bite. Brown recluse spider venom contains enzymes that can lead to local tissue necrosis. This necrosis typically progresses to form an eschar at the center of the bite site. The development of an eschar is a hallmark of necrotic arachnidism, which results from the venom’s effect on the skin and underlying tissues.
Incorrect answers:
B. Gangrene of the distal aspect: A brown recluse bite will be associated with complications near to the bite site. It’s unlikely to cause more distal complications such as gangrene
C. Generalized upper extremity edema: This would suggest a systemic allergic reaction or an infection, neither of which aligns with the isolated and localized nature of the spider bite in this case.
D. Palpable purpuric lesions: Purpura is usually associated with vasculitis or certain infections like meningococcemia, neither of which would develop so quickly following a spider bite.
E. Rapidly spreading erythema: This suggests a bacterial infection, such as cellulitis, which would cause erythema and swelling, often with warmth. However, the pain and progression of the lesion here suggest envenomation rather than infection.
F. Tender, erythematous streaks: This would be consistent with lymphangitis, typically caused by a bacterial infection spreading through the lymphatic system. However, the development of an eschar after a spider bite points more towards necrotic arachnidism than an infection.
Would You Like Us to Help You Ace Your Personal Statement?
Learn More!
Trustpilot
Trustpilot
Question 68:
The correct answer is D. Pericardiocentesis
The patient presents with signs of cardiac tamponade following a penetrating chest wound. The hallmark signs of tamponade, often referred to as Beck’s triad (hypotension, muffled heart sounds, and jugular venous distention), are present due to blood accumulating in the pericardial sac, which increases pressure and impedes cardiac filling. This leads to decreased cardiac output. Pericardiocentesis is the treatment to relieve pressure and stabilize the patient. focused assessment with sonography for trauma (FAST) is helpful in confirming the presence of pericardial fluid in trauma cases.
Incorrect answers:
A. CT scan of the chest and abdomen: Not the immediate choice in an unstable trauma patient. Pericardiocentesis is prioritized to stabilize the patient before imaging.
B. Exploratory laparotomy: This would be indicated for abdominal injuries, but the symptoms suggest cardiac tamponade, not abdominal trauma.
C. Needle decompression and chest tube placement: This is used for tension pneumothorax, which presents differently, with respiratory distress and absent breath sounds, not the cardiac symptoms seen here.
E. Peritoneal lavage: An older method for detecting intra-abdominal injury, now replaced by FAST. In this case, the problem is cardiac, not abdominal.
Question 69:
The correct answer is B. Washing hands thoroughly with soap and water
The patient’s history of Clostridium difficile infection highlights the need for effective hand hygiene. C. difficile spores are resistant to alcohol-based hand sanitizers, so soap and water are necessary to physically remove the spores from the hands. Proper handwashing is crucial to prevent transmission, especially in healthcare settings where the patient has a history of recent infection.
Incorrect answers:
A. Alcohol gel: Alcohol-based hand sanitizers are ineffective against C. difficile spores, which is why soap and water are recommended for hand hygiene in these cases.
C. Surgical mask: C. difficile is not transmitted via respiratory droplets, so a mask is not appropriate for preventing transmission.
D. Sterile gloves and chlorhexidine: This approach is typically used for surgical infections, but the concern here is preventing C. difficile transmission, which is not addressed by these measures.
E. No additional precautions: Given the patient’s recent C. difficile infection, it is essential to take extra precautions to prevent its spread, which includes handwashing.
Question 70:
The correct answer is A. Audiography
The child’s recurrent fractures with minimal trauma and bluish sclera strongly suggest Osteogenesis Imperfecta (OI), a collagen synthesis disorder that causes fragile bones. One common complication of OI is sensorineural hearing loss, which can occur in early childhood or adolescence. Audiography is recommended as part of routine care for children with OI to detect hearing loss early and manage it effectively.
Incorrect answers:
B. DEXA scan: Although OI affects bone strength, DEXA is more useful for diagnosing osteoporosis in adults, especially postmenopausal women, not for diagnosing OI.
C. Echocardiography: OI typically does not involve significant heart abnormalities, so echocardiography is unnecessary unless there are specific cardiovascular symptoms.
D. Retinal examination: There are no primary retinal manifestations of OI, so this exam is not indicated in this case.
E. Serum calcium and vitamin D: While useful for diagnosing metabolic bone diseases or vitamin D deficiency, these tests are not directly related to diagnosing or managing OI.
Don’t let subpar interview performance prevent you from matching. Ace your residency interviews with our experts’ advice! Each session is half real-time mock interview and half as feedback.
Get Our Experts’ Guidance!
Question 71:
The correct answer is E. There is an unclear difference in efficacy between the two drugs at these doses
Methadone and buprenorphine are both effective treatments for opioid use disorder, with proven efficacy in reducing opioid use and relapse rates. Studies comparing their efficacy at medium to high doses have not shown a consistent advantage of one over the other. This patient’s concern about sedation with methadone may favor a transition to buprenorphine, but the decision should also consider her individual preferences, clinical history, and the specific pharmacokinetics of the medications.
Incorrect answers:
A. Clinical evidence does not support a definitive numerical advantage in efficacy between these treatments at medium to high doses.
B. Buprenorphine in combination with benzodiazepines increases the risk of respiratory depression so this statement is incorrect.
C. Both methadone and buprenorphine improve treatment retention, with no conclusive evidence favoring one at medium to high doses.
D. Both are more effective than placebo
Question 72:
The correct answer is E. Inclusion of trials with an intervention duration of 2 weeks
The inclusion of studies with an intervention duration of only 2 weeks undermines the validity of comparing methadone and buprenorphine for opioid maintenance therapy. Opioid use disorder is a chronic condition requiring long-term treatment, and short-term studies fail to capture critical aspects like treatment retention, relapse rates, and sustained abstinence. When evaluating clinical trials, always consider the duration of the study, as shorter trials may not reflect real-world treatment outcomes. In this case, the inclusion of a 2-week trial introduces bias because it does not align with the prolonged treatment course required for opioid maintenance therapy.
Incorrect answers:
A. Exclusion of patients using methadone for acute detoxification: Acute detoxification differs from maintenance therapy, so excluding these patients ensures the study focuses on long-term treatment outcomes.
B. Exclusion of pregnant patients: While this limits generalizability to pregnant individuals, it is often done to minimize risks to the fetus, making it a reasonable exclusion criterion.
C. Inclusion of randomized controlled trials only: RCTs are the gold standard for assessing causality and efficacy, so this strengthens the quality of the evidence.
D. Inclusion of trials that compared active drug with placebo: Comparing active drugs to placebo helps establish efficacy but may not provide insights into head-to-head comparisons, which is the focus here.
E. Use of urinalysis to determine treatment retention: Urinalysis is a reliable, objective measure of abstinence and is commonly used in opioid use disorder studies to assess treatment success.
Question 73:
The correct answer is C. Likelihood that more trials with positive results will be published
Systematic reviews rely on comprehensive data collection, and missing studies with non-significant findings leads to a distorted representation of the intervention’s effects. Publication bias occurs when studies with positive or significant findings are more likely to be published than those with negative or null results. This can lead to overestimating the efficacy of methadone or buprenorphine if systematic reviews or meta-analyses disproportionately include trials with favorable outcomes.
Incorrect answers:
A. Earlier diagnosis of relapse: This would be a detection bias, but nothing in the scenario specifically suggests an earlier diagnosis in one group.
B. Exclusion criteria: While this could introduce bias, the scenario does not provide evidence pointing to it specifically.
D. Source of funding: Although funding can introduce bias, there is no indication in the scenario about the funding source causing bias.
E. Variety of sample sizes: While different sample sizes can influence study results, they do not inherently introduce bias like publication bias.
Crush the Biostatistics of USMLE STEP 1, STEP 2 CK, and STEP 3 exams
Sign up NOW!
Question 74:
The correct answer is E. Pulmonary function testing
The patient’s symptoms of progressive shortness of breath, nonproductive cough, and physical exam findings such as wheezing and prolonged expiratory phase strongly suggest chronic obstructive pulmonary disease (COPD). The first step in diagnosing COPD is spirometry (part of pulmonary function testing), which will confirm the presence of airflow obstruction and help assess the severity of the disease.
Incorrect answers:
A. CT scan of the chest: While useful for assessing various pulmonary conditions, a CT scan is not the first-line diagnostic tool for suspected COPD, as spirometry is more definitive.
B. Echocardiography: This is used to evaluate heart function, not lung conditions like COPD. The patient’s symptoms are more consistent with a pulmonary disorder rather than a cardiac one.
C. Exercise stress testing: Primarily used to evaluate cardiac ischemia or exercise tolerance, not appropriate for diagnosing COPD.
D. PPD skin testing and chest x-ray: These are used for diagnosing tuberculosis (TB). However, the patient’s presentation does not support TB, as she does not have the classic risk factors or symptoms for it.
Question 75:
The correct answer is D. Ultrasonography of the bladder
The patient’s drop in urine output after surgery, combined with bilateral pubic rami fractures, raises concern for a bladder injury or post-renal acute kidney injury (AKI). Ultrasonography of the bladder is the most appropriate next step, as it can quickly detect bladder distension, rupture, or other issues like blood in the bladder, which are common after pelvic fractures. This non-invasive method helps rule out bladder injury as a cause of the patient’s symptoms.
Incorrect answers:
A. CT scan of the abdomen and pelvis: While useful for trauma, it’s not the first step in evaluating the bladder for injury, especially when ultrasonography can provide a faster, more specific answer for bladder-related issues.
B. Repeat surgical exploration of the abdomen: There are no clear signs suggesting an abdominal catastrophe (such as ongoing bleeding or perforation), so repeating the surgery isn’t justified at this point.
C. Transfusion of packed red blood cells: Though the patient has bloody urine, there’s no evidence of active hemorrhage or significant blood loss to warrant immediate transfusion. The issue here is more likely related to urinary retention or injury, not ongoing blood loss.
Question 76:
The correct answer is A. Female factor infertility. The patient’s history of pelvic inflammatory disease (PID) suggests that tubal scarring is a likely cause of her infertility. PID, particularly when untreated or recurrent, can lead to damage to the fallopian tubes, resulting in tubal obstruction or adhesions, which can prevent fertilization. Hysterosalpingography in this case shows absence of contrast in fallopian tubes.
Incorrect answers:
B. Hydatidiform mole: There’s no indication in this case that the patient is at risk for a molar pregnancy. Hydatidiform mole is typically associated with abnormal pregnancies, not infertility in the way described here.
C. Recurrent spontaneous abortions: While PID can increase the risk of ectopic pregnancy, it doesn’t directly cause recurrent spontaneous abortions. This patient’s issue is primarily infertility, likely due to tubal factors, not repeated miscarriages.
D. Successful pregnancy within the next year: Given the patient’s likely tubal damage from PID, the probability of a spontaneous, successful pregnancy is reduced without intervention. A thorough evaluation and potential treatment (e.g., in vitro fertilization) would likely be necessary.
Feeling swamped by the complexities of Systematic Reviews? Kickstart your research journey with our comprehensive Systematic Review Course and build your skills with confidence!
Sign up NOW!
Question 77:
The correct answer is D. Macular degeneration
The patient’s symptoms of distorted vision, where straight lines appear wavy (metamorphopsia), strongly suggest macular degeneration, specifically age-related macular degeneration (AMD). The macula, which is responsible for central vision, degenerates in this condition, leading to characteristic distortion. This presentation is common in older adults and typically involves gradual changes in vision without pain, redness, or photophobia. On the USMLE, recognize that AMD presents with central vision distortion, often affecting tasks like reading or recognizing fine details.
Incorrect answers:
A. Cataracts cause gradual, painless blurring or dimming of vision but no distortion. The lens will appear cloudy
B. Central retinal artery occlusion leads to sudden, painless vision loss, often described as a “curtain” over the eye, without distortion.
C. Closed-angle glaucoma presents acutely with painful vision loss, nausea, and a red eye, which this patient lacks.
E. Temporal arteritis is associated with symptoms like headache and jaw claudication and typically causes sudden, complete vision loss, not distortion.
Question 78:
The correct answer is B. Surgical biopsy of the oral lesion
The patient’s history of long-term chewing tobacco use and the presence of a chronic white lesion on the buccal mucosa suggest leukoplakia, a premalignant lesion. Leukoplakia cannot be scraped off and is associated with an increased risk of dysplasia or carcinoma. A surgical biopsy is the definitive diagnostic step to evaluate for malignancy or dysplasia in such lesions. Smokeless tobacco significantly raises the risk of oral cancers and should always prompt evaluation of suspicious oral findings.
Incorrect answers:
A. Prescribing oral nystatin suspension: Oral nystatin treats oral candidiasis, which presents with white plaques that can be scraped off and typically occurs in immunosuppressed individuals, which is not the case here.
C. Swabbing the lesion for cytology: Cytology is less definitive than a biopsy for detecting dysplasia or carcinoma, especially for suspicious lesions in high-risk individuals.
D. Observation only: Given the patient’s tobacco use and the lesion’s premalignant potential, observation without further evaluation is inappropriate and risky.
Question 79:
The correct answer is E. No further management is indicated at this time
The presence of cerumen (earwax) in the external auditory canals is a normal finding and often serves a protective function. Intervention is unnecessary in asymptomatic patients with no hearing loss, pain, or other complications. Over-manipulation of the ear canal can lead to trauma or infection, so conservative management is appropriate.
Incorrect answers:
A. Irrigation of the ear canals by the physician: Irrigation is unnecessary in asymptomatic patients and could risk complications such as perforation or infection.
B. Manual removal of the cerumen by an otorhinolaryngologist: Reserved for cases where symptoms like hearing loss or pain are present or when visualization of the tympanic membrane is critical.
C. Recommendation for daily use of cotton swabs: Cotton swabs can push the cerumen deeper into the canal, worsening impaction and increasing the risk of trauma or infection.
D. Recommendation for the use of a ceruminolytic agent: Not needed if the patient has no symptoms and does not report hearing difficulty.
Question 80:
The correct answer is C. Small vessel disease
The patient’s sudden onset of left-sided weakness, mild headache, and CT findings consistent with an ischemic stroke in the right putamen are indicative of a lacunar infarct, commonly caused by small vessel disease. Hypertension, a key risk factor, leads to pathological changes in small penetrating arteries, including lipohyalinosis and eventual vessel occlusion. These changes often affect deep brain structures like the basal ganglia, thalamus, and internal capsule. Lacunar infarcts typically result in pure motor or sensory deficits, as seen here.
Incorrect answers:
A. Amyloid angiopathy: Typically causes lobar hemorrhages, especially in elderly patients, rather than ischemic strokes in the basal ganglia.
B. Embolism: Often leads to cortical infarcts with associated cortical signs (e.g., aphasia, neglect), which are absent in this case.
D. Vasculitis: Can cause ischemic strokes but is rare and usually presents with systemic symptoms like fever or multi-organ involvement, which are not present here.
E. Venous thrombosis: More commonly presents with diffuse symptoms like headache, papilledema, or seizures rather than a focal basal ganglia lesion.
Ace Your Step 2 CK with Expert Guidance
Struggling with Step 2 CK preparation? Get personalized 1-on-1 coaching from top scorers and experienced mentors.
Book a Session Today!
Trustpilot
Trustpilot
Block 3
Question 81:
The correct answer is B. This patient, with a medical history of hemophilia A and symptoms of joint pain and swelling, most likely has hemarthrosis (bleeding into the joint). It’s associated with factor VIII deficiency. The primary treatment for hemophilia A is factor VIII replacement therapy. Hemophilia causes deep bleeding (e.g., hemarthrosis), while Von Willebrand disease leads to superficial bleeding (e.g., epistaxis, mucosal bleeding).
Incorrect answers:
A. ADH (Vasopressin) – Used in mild hemophilia A and Von Willebrand disease (VWD) to increase factor VIII and vWF levels, but it is insufficient for moderate to severe hemophilia A or active bleeding.
C. Factor IX concentrate – Used for hemophilia B, which is caused by factor IX deficiency, not hemophilia A.
D. Ferrous sulfate – Used to treat iron deficiency anemia.
E. Fresh frozen plasma (FFP) – Contains clotting factors but is not the first-line treatment for hemophilia A, as it provides less concentrated factor VIII compared to factor VIII concentrate, it’s usually used for rapid reversal of warfarin overdose.
Question 82:
The correct answer is C. Physical documentation provides medico-legal protection for both the physician and the patient. A timeout should be conducted before the surgical procedure to confirm the patient’s name, type of surgery, site, etc., to avoid any medical or surgical errors.
Incorrect answers:
A & B. Anesthesia may affect the patient’s state of consciousness, so asking the patient to recall or sign a new consent form is considered unethical and medically inappropriate.
D. The husband is the next of kin to make medical decisions if the patient loses consciousness as in an emergency situation. However, relying on the husband’s decision regarding the site of surgery is inappropriate.
E. It is inappropriate to proceed with surgery without physical documentation and confirmation via a timeout agreed upon by the surgical team.
Question 83:
The correct answer is B. The patient has 6 weeks of shortness of breath and weight gain, demonstrating manifestations of volume overload, including pulmonary edema (crackles halfway up the lung fields), hepatomegaly, and bilateral lower extremity edema. The presence of an S3 sound is highly specific to heart failure. Given the patient’s history of alcohol use, this is a likely cause of dilated cardiomyopathy. Dilated cardiomyopathy (DCM) can also be caused by viral infections hyperthyroidism, doxorubicin toxicity, and vitamin B1 deficiency. It is characterized by systolic dysfunction, global hypokinesis, and functional mitral regurgitation.
Incorrect answers:
A. Large pericardial effusion could lead to tamponade, which causes hypotension, distant heart sounds, and jugular venous distension. However, the patient’s heart sounds are not distant, and there is an S3 sound.
C. Left ventricular hypertrophy is associated with chronic hypertension. This patient has low blood pressure, and an S4 sound (associated with pressure overload) would be expected, not the S3 (volume overload).
D. Paradoxical septal motion is seen in constrictive pericarditis, which takes months to years to develop. It is associated with a pericardial knock, but the patient has no murmurs.
E. Regional wall motion abnormalities are seen in recent myocardial infarctions (MI). However, the patient has had 6 weeks of symptoms, making MI less likely.
Do You Want to be Part of Our Match Success Stories?
Explore The Match Packages!
Trustpilot
Trustpilot
Question 84:
The correct answer is D. This patient with a history of cystic fibrosis most likely has pneumonia, evidenced by fever, cough, crackles, leukocytosis, and consolidation on the chest X-ray. Sputum culture is the next step to narrow the spectrum of antibiotics and perform sensitivity testing. The most common organisms colonizing the bronchial tree in cystic fibrosis are Staphylococcus aureus in children and Pseudomonas aeruginosa in adults.
Incorrect answers:
A. CT angiography is used for suspected pulmonary embolism (PE). Chest X-ray is typically normal in PE. The presence of leukocytosis, cough, and consolidation supports another diagnosis.
B. Serum immunoglobulin testing is used to diagnose patients with recurrent infections. This patient already has a history of cystic fibrosis complicated by bronchiectasis.
C. Spirometry is used to diagnose obstructive and restrictive lung diseases. The patient’s main presentation is infectious in etiology, so this test may be done later to assess pulmonary function.
E. Transbronchial biopsy is an invasive procedure typically used to diagnose lung cancers or abnormal masses on imaging.
Question 85:
The correct answer is B. This patient has a family history of premature acute coronary events (MI at age 48 years). Fasting serum lipid testing is the next step to screen for familial dyslipidemia. Type II hyperlipidemia, also known as familial hypercholesterolemia, is characterized by mutations in the LDL receptor. Early screening of children, especially those with a family history, is crucial for early detection and management.
Incorrect answers:
A & C. ECG and serum chemistry profile are routine investigations for patients with new hypertension. However, ECG is not indicated in the absence of arrhythmia or structural heart disease. Serum chemistry is used to assess electrolytes, kidney, and liver function.
D. It is inappropriate not to perform further testing given the high-risk family history.
Question 86:
The correct answer is A. This 42-year-old patient with a 6-month history of hot flashes, palpitations, and recent irregular menses most likely has menopausal symptoms. The most sensitive marker for diagnosing menopause is FSH. Low estrogen levels during menopause lead to the loss of negative feedback, resulting in an increase in FSH and LH. Treatment for menopausal symptoms typically involves hormonal replacement therapy (HRT), but SSRIs may be used as an alternative when HRT is contraindicated.
Incorrect answers:
B. Both LH and FSH are elevated in menopause due to the loss of negative feedback by estrogen. However, FSH is a better marker than LH in diagnosing menopause. A high LH relative to FSH could also be seen in polycystic ovarian disease (PCOD).
C. Testosterone is not used to diagnose menopause.
D. ECG is used to diagnose arrhythmia and structural heart diseases. The palpitations the patient developed are most likely attributed to menopause.
E. Pelvic ultrasound is used to diagnose endometrial or ovarian pathologies. This patient has no symptoms suggesting issues with these organs.
Our Clients’ Success Speaks to Our Premium Service!
Our Interview Preparation Packages
Question 87:
The correct answer is B. This picture shows grouped vesicles on the right forearm. The patient developed these skin lesions approximately one day after a picnic where there could be exposure to poison ivy. Contact dermatitis due to poison ivy appears in the areas of contact and takes 24–48 hours to appear (delayed-type IV hypersensitivity reaction). Poison ivy is considered flora.
Incorrect answers:
A & C. Dog allergy and peanut butter could manifest with anaphylaxis such as skin lesions (urticaria) and pulmonary symptoms(wheezes)
D. Perfumed bath soap may cause contact dermatitis, but it will be generalized rather than localized vesicles.
E. Sun rash can cause erythema, redness, and burns. It will be in the sun-exposed areas (face, neck, dorsum of the forearm and hands) and occurs bilaterally, not on one side.
Question 88:
The correct answer is D. This patient has pyelonephritis due to the presence of high fever, elevated WBCs, nitrites, and positive E. coli on urine culture. The next step in managing a pediatric patient with a UTI is performing an ultrasound of the kidneys and bladder to rule out anatomical abnormalities, such as vesicoureteral reflux, which can facilitate the ascent of organisms from the bladder to the kidneys.
Incorrect answers:
A. Ciprofloxacin prophylaxis should be avoided in pediatric patients because it causes cartilage damage. It’s used for GI infections and genitourinary procedure prophylaxis.
B. Clindamycin is usually used for Gram-positive infections (e.g., MRSA); prophylaxis is done after proper diagnosis
C. CT urography is done to assess kidney function and patency of the urinary tract. CT should be avoided in children due to radiation exposure.
E. Voiding cystourethrography is used to diagnose vesicoureteral reflux if suspected based on ultrasound findings (e.g., hydroureter, hydronephrosis).
Would You Like Us to Help You Ace Your STEP Exams?
Learn More!
Trustpilot
Trustpilot
Question 89:
The correct answer is B. This patient most likely has acute mastitis due to the presence of fever, breast tenderness, and erythema. The most common organism is Staphylococcus aureus; dicloxacillin has good coverage against Staph. Continued breast feeding is recommended. Mastitis should be differentiated from Inflammatory breast cancer, the latter is unresponsive to antibiotics and absence of fever.
Incorrect answers:
A. A breast binder increases the risk of milk stasis and can worsen the infection.
C. Fluconazole is used for candida infection, which presents as itching, erythema, and satellite lesions at the breast folds.
D. Fine-needle aspiration is reserved for draining an abscess if there is a fluctuant mass.
E. No further management is indicated is inappropriate, as lack of treatment could lead to complications such as abscess formation.
Question 90:
The correct answer is A. This patient has type II diabetes mellitus, which is associated with adipose tissue resistance to insulin. Consequently, there is a compensatory increase in insulin secretion from the pancreas to overcome peripheral resistance in attempt to lower blood glucose. In the early stages, insulin levels are elevated to overcome peripheral resistance. However, as the condition progresses, the pancreas becomes exhausted, leading to insulin deficiency.
Incorrect answers:
B. Insulin resistance in type II diabetes leads to increased insulin levels, not normal levels.
C. With peripheral hormonal resistance, the body compensates by increasing the hormone level rather than maintaining it at normal levels.
D. Increased insulin levels are expected in type II diabetes to counteract high blood glucose; they are not normal.
E. In a healthy person, glucose, insulin, and insulin receptor responsiveness are all normal.
Question 91:
The correct answer is B. This patient presents with major depressive disorder following a myocardial infarction, evidenced by depressed mood, loss of interest, appetite changes, and insomnia. Depression in the post-MI period not only affects mental health but also reduces physical activity, impairs medication adherence, and may contribute to physiological changes (e.g., increased inflammation and autonomic imbalance) that worsen cardiac outcomes. As a result, depression in patients with a history of MI is associated with a doubled risk of death during the subsequent 3 months.
Incorrect answers:
A. Depression decreases quality of life and significantly impacts mortality.
C. Although resting and avoiding socializing might seem beneficial, these behaviors impair both physical and mental health, potentially leading to weight gain and autonomic imbalance, which can further worsen coronary outcomes.
D. Depression has a significant effect on physical health and is associated with worse outcomes, not an absence of impact.
Become a Researcher in the United States
No Experience Required! Just join the live & interactive workshop to learn all the secrets of landing a research position in the U.S
Enroll Here
Trustpilot
Trustpilot
Question 92:
The correct answer is E. This patient with adult-onset immunodeficiency due to recurrent bacterial pneumonia over the past 5 years and decreased levels of IgA, IgG, and IgM likely has common variable immunodeficiency(CVID). Management includes monthly immune globulin replacement therapy to decrease the risk of recurrent infections. CVID patients are at high risk of developing celiac disease, bronchiectasis and lymphoma.
Incorrect answers:
A. Daily inhaled tobramycin is effective against gram-negative bacteria (e.g., Pseudomonas) in cystic fibrosis.
B. Daily intranasal glucocorticoid therapy is effective for allergic rhinitis/sinusitis.
C. Daily TMP/SMX is prophylactic against Pneumocystis jirovecii pneumonia(PCP) in HIV.
D. Intranasal influenza virus vaccine does not address the underlying immunoglobulin deficiency, though it is beneficial as part of routine vaccinations.
Question 93:
The correct answer is A. This patient on warfarin therapy has an elevated PT/INR, likely due to the recent addition of TMP/SMX, which inhibits the CYP450 system, warfarin metabolism is decreased, resulting in an increased plasma concentration. Other medications that inhibit the P450 system include: Cimetidine, Ciprofloxacin, Omeprazole, Metronidazole, Erythromycin, protease inhibitor, grapefruit (COKE,PIE and grapefruit)
Incorrect answers:
B. Certain vitamin K-rich foods (e.g., spinach) can decrease INR, not increase it.
C. Echocardiography is used to assess structural heart disease but is not relevant here.
D. Venous duplex ultrasound is indicated for suspected deep vein thrombosis (e.g., unilateral leg swelling, edema), which is not the case here.
E. Allopurinol does not have a significant interaction with warfarin.
Question 94:
The correct answer is C. This patient with colon cancer has a history of bowel obstruction and prior bypass surgery. Given that he is not a candidate for further surgical intervention, another operation would not be beneficial and could lead to complications. The best approach is to clearly explain the risks and futility of surgery to the patient and his family.
Incorrect answers:
A. While patient autonomy is important, Nonmaleficence sometimes takes precedence when an intervention is unlikely to provide benefit and may cause harm.
B. Altering chemotherapy is unlikely to resolve the current obstruction.
D. Transferring the patient to another facility would not change the prognosis or treatment options.
E. External beam radiation is more beneficial for localized metastatic prostate cancer to the spine, not for diffuse peritoneal carcinomatosis.
Get access to our list of Electives and U.S. Clinical Experience for IMGs!
Learn More
Question 95:
The correct answer is C. This patient presents with chronic malaise, fatigue, cough, low-grade fever (37.7°C), and a reticulonodular pattern on chest X-ray, which is characteristic of hypersensitivity pneumonitis (HP). HP is often caused by exposure to environmental antigens, such as wood dust in this scenario. Treatment is usually avoidance and elimination of antigen, corticosteroid for acute symptoms. Chronic disease requires immunosuppressive therapy.
Incorrect answers:
A. Aspergillosis: Aspergillus can cause aspergilloma, which appears as a round mass on chest X-ray, or allergic bronchopulmonary aspergillosis, but the reticulonodular pattern makes aspergillosis less likely.
B. Asthma recurrence: Asthma exacerbations are usually associated with an obstructive pattern on pulmonary function tests, while this patient shows a mixed obstructive and restrictive pattern. Additionally, chest X-ray would show hyperinflation, not a reticulonodular pattern.
D. Sarcoidosis: Sarcoidosis is more common in middle-aged African American women and typically presents with interstitial infiltrates and hilar lymphadenopathy
E. Silicosis: Silicosis is common in workers exposed to silica dust, and while it typically affects the upper lung fields, it would also show characteristic hilar lymph node calcifications (egg shell calcifications).
Question 96:
The correct answer is A. This patient presents with progressive shortness of breath and fine crackles, which are suggestive of pulmonary fibrosis likely due to Amiodarone. Management is typically discontinuation of amiodarone and corticosteroids for acute symptoms.
Incorrect answers:
B. Lisinopril: While lisinopril can cause a dry cough, it does not lead to pulmonary fibrosis.
C. Propranolol: Although propranolol may exacerbate symptoms in patients with COPD, it is not associated with pulmonary fibrosis.
D. Tiotropium: Tiotropium is an anticholinergic medication, and its side effects primarily include dry mouth.
E. Warfarin: Warfarin’s primary side effect is bleeding. It may interact with amiodarone and increase bleeding.
Question 97:
The correct answer is E. The physician likely appears intoxicated due to alcohol, which impairs their ability to provide safe patient care. Ensuring patient safety is the priority, and the physician should be removed from the clinic to prevent any harm to patients. For USMLE, the principle of patient safety takes precedence over physician well-being is emphasized.
Incorrect answers:
A. Allowing the physician to continue working would potentially harm patients. Immediate action should be taken to prevent this.
B. Cancelling the remainder of the clinic hours would disrupt patient care, but it does not address the intoxication issue directly.
C. Asking the physician assistant to examine patients is not an appropriate response because the physician’s intoxication needs to be addressed directly.
D. Confronting the physician may not guarantee they leave, and it doesn’t immediately address the risk to patient safety.
F. Reporting to the state medical board is a more serious step and should be reserved for gross negligence or unethical conduct. Immediate action in the clinic is required first.
What Do Our MATCH Application Packages Include?
Advisor UNLIMITED Access
We get how stressful the residency match process is, so we're here for you - communicate with your personal advisor ANYTIME you need!
Personal Statement Editing
Our editing includes not only language but also context, structure, and content advising.
ERAS Application Editing
The editing goes beyond language and grammar corrections to structure, design, and content based on your personal story and achievement.
Interview Preparation
The best way to learn something is to do it. That’s why we divide our interview preparation sessions into two parts.
Mock Interview + Feedback
Sign up NOW!
Question 98:
The correct answer is E. This patient has a unicornuate uterus, a congenital anomaly that can be associated with other congenital conditions, such as having a single kidney (as in this case). A single kidney typically doesn’t pose significant risks during pregnancy. Abnormality in the structure of uterus (e.g. cornuate uterus, fibroids) raises the risk of preterm labor.
Incorrect answers:
A. Fetal macrosomia: Fetal macrosomia is generally associated with diabetes and obesity
B. Multiple gestation: Multiple gestation is more likely due to genetic predisposition or medication use (e.g., clomiphene).
C. Oligohydramnios: A single kidney is unlikely to cause oligohydramnios, which is typically related to fetal renal issues.
D. Preeclampsia: A single kidney is not associated with an increased risk of preeclampsia.
Question 99:
The correct answer is C. This 62-year-old woman with bipolar disorder presents with confusion and a recent tonic-clonic seizure. Her symptoms, including confusion, seizure, and a second-degree atrioventricular block, are likely due to lithium toxicity. It has a narrow therapeutic index. Factors like dehydration, as seen in this patient can decrease renal clearance of lithium, leading to toxicity. Lithium toxicity can also occur with NSAIDs and diuretics.
Incorrect answers:
A. Acetaminophen: Acetaminophen overdose leads to hepatotoxicity, not confusion or heart block.
B. Bupropion: While bupropion can lower the seizure threshold, it does not typically cause heart block or confusion.
D. Risperidone: This antipsychotic can cause QT prolongation and extrapyramidal symptoms, but not heart block or confusion.
E. Topiramate: While topiramate is linked to kidney stones, it does not typically cause confusion or heart block.
Question 100:
The correct answer is E. This child is undergoing a dental procedure and has no risk factors that would warrant antibiotic prophylaxis for bacterial endocarditis. A heart murmur alone is not considered high risk for prophylaxis. For USMLE, Systolic murmur in young asymptomatic people are usually innocent murmur, no further investigation or management should be done. Diastolic murmur is always pathologic
Incorrect answers:
A. Azithromycin: Typically prescribed for community-acquired pneumonia, not for endocarditis prophylaxis in this scenario.
B. Cephalexin: An alternative to amoxicillin for mild allergic reactions, but unnecessary in this case.
C & D. Clindamycin and Vancomycin: Alternatives to amoxicillin for severe allergic reactions, but not required here due to the lack of high-risk factors.
Boost your USMLE scores with personalized, one-on-one tutoring from our expert tutors at The Match Guy!
Learn More!
Trustpilot
Trustpilot
Question 101:
The correct answer is A. This patient has oligospermia and is experiencing infertility. The most appropriate initial action is to assess reversible causes of oligospermia, such as alcohol consumption, smoking, and stress levels. These factors can negatively affect sperm count and quality and should be addressed before considering more invasive treatments. Always consider reversible causes before attempting any medical or invasive procedures.
Incorrect answers:
B. At 30 years old, a decline in sperm count due to age is unlikely to be significant.
C. Phosphodiesterase inhibitors are used to treat erectile dysfunction, not oligospermia.
D. Testosterone replacement can suppress sperm production, potentially worsening infertility.
E. Reversible causes should be explored and addressed first.
Question 102:
The correct answer is B. This patient has an itchy, burning rash with clear vesicles that rupture easily, coinciding with recent exposure to high temperatures during a vacation. These findings are consistent with miliaria crystallina (heat rash). Avoiding heat and keeping the skin cool is the best management. The rash resolves spontaneously without the need for medical treatment.
Incorrect answers:
A. Antifungal cream: Pityriasis versicolor can appear in warm weather but presents with pigmented patches, not vesicles or burning pain.
C. Oral antibiotic therapy: There is no indication of a bacterial infection requiring antibiotics.
D. Oral antiviral therapy: Varicella-zoster virus (chickenpox) causes vesicles at different stages of healing with crusting, which is not seen in this case. It’s usually surrounded by erythema.
E. Oral retinoid therapy: Retinoids are used for acne and are not appropriate for heat rash.
F. Ultraviolet therapy: UV therapy is used for conditions such as psoriasis but is not beneficial for heat rash.
Question 103:
The correct answer is E. Thyrotoxicosis. This patient has classic signs of hyperthyroidism, including anxiety, insomnia, increased bowel movements, weight loss, tremors, and nail separation. The diffusely enlarged thyroid with a bruit suggests Graves’ disease, Additionally, her irregularly irregular pulse indicates atrial fibrillation, a known complication of hyperthyroidism. For USMLE, any patient with recent onset atrial fibrillation, do TSH testing.
Incorrect answers:
A. Addison disease: Typically presents with fatigue, abdominal pain, weight loss, and hypotension—not hypertension and atrial fibrillation.
B. Carcinoid syndrome: Can cause frequent diarrhea, but it is usually accompanied by flushing and wheezing, which are absent here.
C. Cushing syndrome: May present with hypertension, but patients usually have weight gain, striae, and a buffalo hump, which are not seen in this case.
D. Pheochromocytoma: Causes episodic headaches, palpitations, and paroxysmal hypertension, whereas this patient’s symptoms are persistent, making it unlikely.
Would You Like Us to Help You Ace Your Personal Statement?
Learn More!
Trustpilot
Trustpilot
Question 104:
The correct answer is B. Absolute risk reduction (ARR) is calculated as:
ARR=Risk in Placebo−Risk in SERM=3.6−1=2.6
Incorrect answers:
A, D & E. Incorrect values.
C. 3.6 represents the relative risk of invasive breast cancer in the placebo group, not the ARR.
Question 105:
The correct answer is E. This patient has knee pain with a limp and stable bone polyps on x-ray, unchanged from 6 months ago, consistent with multiple osteochondromas. As these are benign lesions without concerning features, no further workup is needed, and the patient can bear weight as tolerated. For bone lesions, if there’s no invasion, significant pain or new growth, it’s unlikely to be malignant.
Incorrect answers:
A, B, C & D. Osteochondromas are benign, and no malignant changes are present. MRI or biopsy is unnecessary unless there is suspicion for malignancy (e.g., new growth, pain at rest).
F. Non-weight-bearing is indicated only in cases of joint instability, which is not present here.
Question 106:
The correct answer is B. Adoption should be discussed as early as possible, even if the child cannot fully understand. This fosters mutual trust, emotional security, and a strong sense of self-identity. For USMLE, emphasizing open communication and encouraging patients to discuss important information is key, as it aligns with patient-centered care
Incorrect answers:
A. Waiting until adulthood can cause mistrust and emotional distress.
C. Adoption records are not a requirement for when to discuss adoption.
D. There is no strict age; earlier discussions are preferable.
E. Delaying until the child discovers the truth independently can lead to emotional turmoil and trust issues.
Don’t let subpar interview performance prevent you from matching. Ace your residency interviews with our experts’ advice! Each session is half real-time mock interview and half as feedback.
Get Our Experts’ Guidance!
Question 107:
The correct answer is B. This patient presents with acute onset of severe left foot pain, swelling of the big toe, and an elevated serum uric acid level, which are typical features of gout. The inability to walk and the lack of trauma support the diagnosis. Serum uric acid levels, while not diagnostic by themselves, are commonly elevated in gout, though they can be normal or even decreased during an acute attack. In addition, her creatinine level of 1.4 mg/dL suggests some renal impairment, which could contribute to decreased excretion of uric acid and precipitate a gout flare. Management include NSAIDs (avoid in kidney injury), Colchicine and corticosteroids
Incorrect answers:
A. it typically presents with fever, erythema, warmth, and diffuse skin swelling, often with purulent drainage
C. This condition causes milder symptoms and usually affects the wrist, not the big toe.
D. it Generally affects weight-bearing joints like the knees or hips, with chronic pain exacerbated by activity.
E. it’s Typically associated with conditions like hemochromatosis, pseudogout primarily affects larger joints such as the knee
F. it affects multiple small joints, especially in the hands and wrists, with a chronic course. It is less likely to present acutely in the big toe, and there is no history of rheumatoid arthritis here
Question 108:
The correct answer is D. Bacterial vaginosis (BV) is indicated by the patient’s white-gray discharge and elevated vaginal pH (normal is 3.8-4.5). BV is a known risk factor for postoperative infections after gynecologic procedures. BV is NOT sexually transmitted but it’s associated with sexual activity. No need to treat the male partner, unlike Trichomonas vaginalis infection.
Incorrect answers:
A. History of upper respiratory tract infection: This is not directly linked to pelvic infections or postoperative complications.
B. HSV 2: While HSV 2 can cause complications in the presence of active lesions, this patient has no active lesions, so it is not the strongest risk factor here.
C. Obesity: Obesity can lead to worsened postoperative outcomes, but it is not the primary concern in this context.
E. Vulvar shaving: Shaving can introduce minor skin irritation or tears, but it is not as significant a risk factor for infection as BV.
Question 109:
The correct answer is B. This patient most likely has Lyme disease, evidenced by Bell’s palsy (unilateral facial weakness) and meningitis symptoms (neck flexion induces pain, lymphocytic predominance on lumbar puncture). The second stage of Lyme disease can cause facial nerve palsy. Lumbar puncture in Lyme is similar to viral infection. Also in Lyme arthritis, there’s lymphocytic predominance, WBCs count is around 20,000.
Incorrect answers:
A. Measurement of ACE activity could be associated with sarcoidosis. The common presentation is a middle-aged African American woman with chronic cough and hilar adenopathy on chest X-ray.
C. CMV can cause mononucleosis-like symptoms, including fever, fatigue, and tonsillar exudates.
D. Serum antinuclear antibody is used in suspected rheumatologic conditions like SLE; the absence of malar rash and joint pain makes it less likely.
E. Serum protein electrophoresis is used in the diagnosis of multiple myeloma, which is characterized by hypercalcemia, bone pain, kidney injury, and anemia.
Question 110:
The correct answer is D. This patient has a chronic history of fatigue and laboratory findings showing an elevated WBC count with neutrophilic predominance, along with the presence of metamyelocytes and myelocytes, which supports the diagnosis of chronic myeloid leukemia. For the USMLE, the presence of blast cells supports the diagnosis of AML or ALL, while the presence of –Cytes supports the diagnosis of or CLL.
Incorrect answers:
A. Acute lymphocytic leukemia is more common in children and typically presents with anemia, thrombocytopenia, and increased lymphoblasts on a peripheral blood smear.
B. Acute myeloid leukemia is characterized by an increase in myeloblasts, not myelocytes.
C. Chronic lymphocytic leukemia is usually asymptomatic in elderly patients and shows lymphocytic predominance, not myelocytic.
E. Monoclonal gammopathy of undetermined significance is characterized by <10% clonal plasma cells on bone marrow biopsy.
A comprehensive guide to crush the Biostatistics of USMLE STEP 2 CK exam!
Sign up NOW!
Question 111:
The correct answer is D. This neonate with myelomeningocele and loss of movement in the lower extremities likely has significant spinal cord injury in the lumbosacral region. Neurogenic bladder dysfunction is common due to pelvic nerve involvement. The lower extremity and bladder are innervated by the same spinal and cortical regions, which is why dysfunction in these areas often occurs together.
Incorrect answers:
A. Congenital heart malformations can occur in isolation or as part of syndromes such as velocardiofacial syndrome. However, they are not directly linked to myelomeningocele.
B. Cryptorchidism is more commonly associated with prematurity or testosterone deficiency rather than neural tube defects.
C. Spina bifida is often due to inadequate maternal folate intake, but it is not directly associated with impaired folate metabolism.
E. Although mild ventriculomegaly is present, it does not necessarily indicate severe cognitive impairment.
Question 112:
The correct answer is E. This adolescent, who has recently started socializing with new peers and engaging in minor rule-breaking behavior, is likely exhibiting normal development. At this age, adolescents naturally seek independence and privacy while forming stronger peer relationships. For psychiatry questions, if a patient develops behaviors that are not extreme and do not cause significant occupational or social problems, they are typically considered within the range of normal behavior.
Incorrect answers:
A. Adjustment disorder occurs in response to an identifiable stressor and causes significant distress or impairment, which is not evident in this case.
B. ADHD is characterized by attention deficits and hyperactivity in multiple settings (e.g., home and school), typically present from early childhood.
C. Conduct disorder involves stealing, physical aggression, and violating others’ rights, which are not observed in this patient.
D. Oppositional defiant disorder is characterized by vindictiveness and defiant behavior without physical aggression or severe rule-breaking.
Question 113:
The correct answer is A. Conducting weekly structured team briefings and daily huddles is the most effective method for maintaining a common mental model and ensuring clear communication within the team. Regular meetings provide an opportunity to share updates on patients and adjust care plans as needed, fostering a shared understanding of patient status and goals. For safety and quality questions, it’s important to focus on measures that are efficient and resource-effective, avoiding time-consuming processes or financial compensations (such as paying doctors and nurses) that could lead to wasted resources.
Incorrect answers:
B. Educating team members on protocols for common illnesses is important for clinical practice but does not directly improve communication within the team or ensure a shared understanding of individual patient cases.
C. Discussing the case with each team member individually may lead to miscommunication and lack of cohesion, rather than promoting effective team communication.
D. While maintaining a consistent team of intensivists can improve continuity of care, it does not specifically address communication between team members.
E. A disease-specific ICU may improve care for particular conditions but does not inherently enhance communication among the broader team caring for complex patients.
Question 114:
The correct answer is D.This patient presents with symptoms resembling mononucleosis (fever, muscle ache, fatigue, sore throat), and the presence of leukopenia combined with a negative monospot test suggests HIV infection as the most likely diagnosis. For USMLE, mononucleosis like symptoms, suspect EBV, CMV, Toxoplasmosis and HIV.
Incorrect answers:
A. CMV infection typically affects immunocompromised patients or those who have undergone solid organ transplants. It’s not associated with leukopenia
B. Monospot test is highly sensitive for detecting EBV. A negative monospot test effectively rules out EBV infection
C. Hemagglutinin testing is used for Mycoplasma pneumoniae infections, which commonly cause subacute symptoms such as cough and fatigue. Mycoplasma infection may lead to anemia but does not typically present with leukopenia.
E. Hepatitis B virus infection is more commonly associated with symptoms such as malaise, joint pain, and rash, resembling serum sickness. It is unlikely to cause leukopenia.
Would You Like Us to Help You Ace Your STEP Exams?
Learn More!
Trustpilot
Trustpilot
Question 115:
The correct answer is E. The patient has declining mental status and likely lacks the capacity to make medical decisions. In the absence of durable power of attorney, the next of kin is responsible for making medical decisions. In this case, the son is the next of kin. Next-of-kin decision makers are: 1) advance directive, 2) Spouse, 3) adult children, 4) parents, 5) adult siblings
Incorrect answers:
A. Boyfriend: Without legal marriage or a designated healthcare proxy, the boyfriend is not considered the next of kin.
B. Neighbor: While the neighbor is a trusted individual, she is not a family member and is at the lowest level of the decision-making hierarchy.
C. Patient: The patient’s declining mental status makes it ethically inappropriate for her to make medical decisions.
D. Physician: The physician cannot make medical decisions, except in emergency situations when no family members are available or when a substituted decision is necessary.
Question 116:
The correct answer is C. This patient presents with symptoms of acute appendicitis (fever, migratory abdominal pain, right lower quadrant tenderness, and leukocytosis) and an Alvarado score of 6, which falls into the moderate-risk category. In this case, CT is the most appropriate next step for further assessment, ruling out other similar presentations (e.g. Crohns disease) and surgical decision making.
Incorrect answers:
A. Ultrasound is preferred in young children and pregnant women because it avoids radiation exposure.
B. This is used to diagnose and treat intussusception, which typically presents with intermittent abdominal pain. This patient’s symptoms are more consistent with acute appendicitis.
D. This is primarily used to diagnose Meckel’s diverticulum, which is a congenital anomaly where there are ectopic gastric acid-secreting cells.
E. This test is used for diagnosing intestinal obstruction in the upper GI tract (eg. due to adhesions, tumors, or hernias).
Question 117:
The correct answer is C. Diverticulosis. The patient’s painless, bright red rectal bleeding, especially in the absence of other significant gastrointestinal symptoms, is consistent with diverticulosis. The use of NSAIDs, which can increase the risk of diverticular bleeding, is common in elderly patients. Management include increasing fibers intake, hydration, avoid red meat.
Incorrect answers:
A. Anal fissures are typically associated with tearing pain, often during bowel movements, and not painless rectal bleeding. They are also more common in patients with chronic constipation.
B. Although colonic polyps can cause rectal bleeding, this patient had a colonoscopy 8 years ago with normal findings, making it less likely.
D. Duodenal ulcers typically cause melena (dark, tarry stools) due to upper gastrointestinal bleeding, not fresh, bright red blood.
E. Gastritis generally presents with symptoms like epigastric pain or discomfort and results in melena or coffee-ground emesis, not fresh red blood in the stool.
F. Ulcerative colitis typically affects younger patients and presents with bloody diarrhea and other inflammatory symptoms (fever, abdominal pain), which are not seen in this case.
Question 118:
The correct answer is C. This child with a recent ventricular septal defect (VSD) repair has developed decompensated heart failure, evidenced by cool, clammy skin, cold distal extremities, delayed capillary refill, and minimal urine output (suggesting decreased perfusion). Echocardiography is the most appropriate next step as it is noninvasive and readily available.
Incorrect answers:
A. This is an invasive procedure and is not recommended for the evaluation of decompensated heart failure.
B. It carries risks such as radiation exposure and contrast-induced nephrotoxicity, making it less ideal for this situation.
D. Electrophysiology study: It is used to diagnose arrhythmias, but the patient’s ECG shows no significant findings.
E. MRI of the heart: This is time-consuming, requires sedation, and is not the best first-line evaluation for decompensated heart failure.
Question 119:
The correct answer is E. This patient with frequent light-headedness, relieved by eating snacks, and a serum glucose level of 41 mg/dL likely has hypoglycemia due to high insulin levels. Normally, insulin and C-peptide are secreted in proportionate amounts. However, with external insulin administration, endogenous insulin production is suppressed, leading to elevated insulin levels and low C-peptide levels
Incorrect answers:
A. Carcinoid syndrome: This syndrome is associated with the triad of diarrhea, wheezing, and flushing, not hypoglycemia.
B. Pseudopancreatic cyst: This is a complication of acute pancreatitis, not related to insulin levels.
C. Pancreatic adenocarcinoma: A mass in the head of the pancreas is often suggestive of pancreatic adenocarcinoma, which typically presents with painless jaundice.
D. Insulinoma: A vascular mass in the neck of the pancreas could suggest insulinoma, which is associated with elevated insulin and C-peptide levels, not low C-peptide.
Question 120:
The correct answer is D. This patient presents with rapidly progressing right lower extremity edema, cellulitis, and hemorrhagic bullae following a minor scratch. His worsening pain, tachycardia, and hypotension strongly suggest necrotizing fasciitis, a life-threatening soft tissue infection. Immediate surgical debridement is essential to remove necrotic tissue, prevent further progression, and improve survival.
Incorrect answers:
A. Draining the bullae is not the priority; necrotic tissue must be surgically removed.
B. May be beneficial as adjunctive therapy but should not delay surgery
C. Imaging is time-consuming and unlikely to alter the urgent need for debridement, as the diagnosis is clinically evident.
E. Delaying intervention risks worsening necrosis, sepsis, and limb loss.
Want Us to Help You Get Into Residency?
Schedule Your Free Consultation
Trustpilot
Trustpilot
That brings us to the end of the new Free 120 NBME Step 2 sample test answers and explanations. We hope this breakdown helped you better understand the reasoning behind the correct answers and why the other options were incorrect. Hopefully, this will give you an edge in acing your USMLE Step 2 exam!
Disclaimer: We did our absolute best to make sure everything is accurate and double checked everything but as with anything done by a human there is still potential for mistakes so if you find anything, please let us know by emailing us at info@thematchguy.com
For personalized Step 2 CK preparation strategies, high-yield resources, and expert tutoring, explore our Proven USMLE Tutoring packages here.
If you need guidance on your target USMLE scores or have questions on improving your chances of matching into your dream specialty, feel free to schedule a call here or text us on WhatsApp here for personalized support.
🍀 We wish you best of luck on your exam!
Latest Articles
Step 1 Sample Test Answers + Explanations 2025 | USMLE STEP 1 Free 120 Explanations
Malke AsaadFebruary 21, 2025
Step 2 Sample Test Answers + Explanations 2025 | USMLE STEP 2 Free 120 Explanations
Malke AsaadFebruary 21, 2025
SOAP for MATCH 2025: A Step-by-Step Guide for Unmatched Residency Applicants
Malke AsaadFebruary 11, 2025
USMLE Step 1 and Step 2 CK Average Scores by Specialty 2025
Malke AsaadFebruary 10, 2025
Need Help?
About TheMatchGuy
TheMatchGuy is a premier medical consulting firm trusted by over 35,000 students and doctors worldwide. Known for our track record and specialization in helping MATCH® Applicants secure residency spots at programs in the United States 🇺🇸 and exceptional guidance in Research, USMLE prep, and clinical experience services. Learn more about how they can help you achieve your career goals. Schedule a discovery call for FREE now.
How We Can Help You
Trustpilot